XI Открытый Всероссийский Синхронный Чемпионат. Результаты по вопросам




1. Вопрос 4.35:

Раздаточный материал

    Hot Hot Searing! Ouch Ouch Ouch Ouch!

    [Ведущему: раздаточный материал не зачитывать].
    Горячо, горячо, обжигает! Ай Ай Ай Ай! Назовите двумя словами то, чему по-священ розданный вам текст.

Ответ: Серная кислота.

Комментарий: Первые буквы фразы составляют химическую формулу серной кислоты - H2S04.

Источник: http://uncyclomedia.org/wiki/UnSource:List_of_acronyms

Автор: Сергей Лобачев (Нижний Новгород).

Средняя оценка: 1.72

Комментарии


2. Вопрос 5.9:

Раздаточный материал

    Предусмотрительные непальские гонцы несли два послания сразу: письменное и устное

    Предусмотрительные тибетские гонцы несли два послания сразу: письменное и устное. Какие слова пропущены в предыдущем предложении?

Ответ: лживое, правдивое.

Зачёт: вместо лживого – фальшивое, ложное, ненастоящее, фиктивное. Вместо правдивого – настоящее, правильное, подлинное, истинное. В любом порядке.

Комментарий: Письменное лживое и устное правдивое. В письменной раздаче мы вам тоже соврали, сказав, что гонцы были непальскими.

Источник: Чернышёв Д.А. Как люди думают. — М.: Манн, Иванов и Фербер, 2013. — с. 157.

Автор: Алексей Бороненко (Челябинск — Москва).

Средняя оценка: 1.18

Комментарии


3. Вопрос 1.24: В книге «Сто великих памятников» напряженную фигуру Геракла сравнивают с НЕЙ. При этом ЕЁ саму изваять не удалось. Назовите ЕЁ.

Ответ: тетива.

Зачёт: точный ответ.

Комментарий: Геракл изображен натягивающим лук.

Источник: 1)http://www.many-books.org/auth/3398/book/62521/samin_d_k/100_velikih_pamyatnikov/read/79

Автор: Константин Костенко (Гремячинск).

Средняя оценка: 1.02

Комментарии


4. Вопрос 1.25: Одна компания представила набор, состоящий из пяти баночек, соответствующих злости, смеху, луку, чиханию и печали. Назовите и пищевой продукт в этих баночках, и сырьё, по утверждению компании, применявшееся для его производства.

Ответ: соль, слёзы.

Зачёт: точный ответ в любом порядке.

Комментарий: набор соли из человеческих слез.

Источник: http://www.etoday.ru/2013/03/banochki-s-sol-yu-iz-slez-ot-h.php

Автор: Александр Пономарев (Пермь).

Средняя оценка: 1.01

Комментарии


5. Вопрос 2.30: Люксембургский спортсмен ЖозИ БартЕль, завоевавший олимпийское золото в 1952 году, расплакался на пьедестале, поскольку услышал ЕЁ. Марк Твен признавался, что хорошую ЕЁ готовил три дня. Назовите ЕЁ одним словом.

Ответ: Импровизация

Зачёт: Точный ответ

Комментарий: Гимн Люксембурга не смогли найти, в итоге мелодия гимна была сымпровизирована группой музыкантов – настолько неудачно, что стоящему на пьедестале чемпиону не оставалось ничего, кроме как обхватить голову руками и заплакать.

Источник: 1) «99 фактов об Олимпиаде» 2) http://citaty.info/tema/improvizaciya 3) http://ru.wikipedia.org/wiki/Бартель,_Жози

Автор: Андрей Грамматик (Николаев, Украина).

Средняя оценка: 1

Комментарии


6. Вопрос 3.10: В статье о крупных кожистых черепахах и рыбах-прилипалах рассказывается о своеобразном ИКСЕ. К мастерам ИКСА в шутку относят Михаила Ломоносова. Назовите ИКС одним словом.

Ответ: автостоп.

Зачёт: хитчхайкинг.

Комментарий: рыбы-прилипалы цепляются к черепахе и на ней перемещаются под водой. Ломоносов, как известно, добирался в Москву на попутном возе с рыбой.

Источник:
    1. http://nat-geo.ru/article/724-chempionyi-po-vyizhivaniyu/
    2. http://ru.wikipedia.org/wiki/Автостоп

Автор: Артём Викторов (Саратов)

Средняя оценка: 0.95

Комментарии


7. Вопрос 1.33: Лезгинская пословица гласит, что два представителя этой специальности не могут работать вместе - ведь тогда они не смогут разойтись. В биографии одного из них упоминаются балалайка в руках и даже мешок на голове. Назовите эту специальность словом с двумя корнями.

Ответ: канатоходец.

Зачёт: точный ответ.

Комментарий: Два канатоходца не могут ходить по одному канату. Канатоходец Федор Молодцов ходил и с балалайкой в руках, и с мешком на голове.

Источник: 1)http://uznayslovo.ru/sarticles.php?id_cat=283 2)http://fisechko.ru/100vel/fenomin/78.html

Автор: Константин Костенко (Гремячинск).

Средняя оценка: 0.94

Комментарии (1)


8. Вопрос 4.10:

Раздаточный материал

    crazed zebras craved egress
    at a garage
    scared bats vacated
    watered carafe
    begat a gaffe
    at a cafe

    [Ведущему: раздаточный материал не зачитывать].
    Писатель Уильям Зинсер сочинил необычное произведение для своего друга, перенёс-шего инсульт. Данное произведение получило название – «фантазия для НЕЁ». Назовите ЕЁ двумя словами.

Ответ: Левая рука.

Комментарий: Данное произведение - своего рода восстановительное упражнение, в ко-тором все слова состоят исключительно из букв, находящихся на левой части клавиату-ры.

Источник: http://www.futilitycloset.com/2013/10/19/side-matters/

Автор: Сергей Лобачев (Нижний Новгород).

Средняя оценка: 0.9

Комментарии


9. Вопрос 1.21: Сигареты марки «Полёт» одно время были упакованы в пачке плотнее, чем другие сигареты и согласно надписи на пачке назывались ТАКИМИ. В каком городе в 1909 году появилось помещение, называемое ТАКИМ?

Ответ: Вашингтон.

Зачёт: точный ответ.

Комментарий: ТАКИМИ – овальными. В пачке был еще один ряд сигарет. Помещение - Овальный кабинет в Белом доме.

Источник: 1)http://ussr365.blogspot.ru/p/blog-page_3342.html 2)http://en.wikipedia.org/wiki/Oval_Office

Автор: Константин Костенко (Гремячинск).

Средняя оценка: 0.84

Комментарии


10. Вопрос 5.10: В сатире времён Первой мировой войны описан обычай, на который немецкие семьи отводили каждое утро немного времени. Кто, согласно одной статье, использовал впоследствии это описание?

Ответ: Джордж Оруэлл.

Зачёт: Оруэлл, Эрик Артур Блэр.

Комментарий: сатирики сообщали, что немецкие семьи собирались за кухонным столом для «утренней ненависти» в адрес англичан. В виде «двухминутки ненависти» это выражение попало в роман «1984».

Источник: http://en.wikipedia.org/wiki/Two_Minutes_Hate

Автор: Алексей Бороненко (Челябинск — Москва).

Средняя оценка: 0.83

Комментарии


11. Вопрос 1.17: В Вопросе есть замена. По предположению Шермана Зильбера, изменения климата могли послужить причиной того, что после определенного момента у подавляющего большинства динозавров был один и тот же ИГРЕК. А какой именно ИГРЕК?

Ответ: мужской.

Зачёт: самец, мужчина, синонимичные ответы.

Комментарий: По версии Зильбера, динозавры вымерли от нехватки самок. Если бы мало было самцов, то они бы, вероятно, справились.

Источник: http://www.publy.ru/post/5674

Автор: Александр Пономарев (Пермь).

Средняя оценка: 0.81

Комментарии


12. Вопрос 2.6: Донесения советских разведчиков далеко не всегда были точными и достоверными. В одной статье по этому поводу говорится, что посещения ИХ принесли бы существенно больше точной информации о планах Третьего Рейха, нежели донесения разведки. Произошедшему в одной из НИХ событию посвящена мемориальная доска, на которой можно прочитать имена четырех полицейских. Назовите это событие.

Ответ: Пивной путч

Зачёт: По смыслу, например, путч 1923 года / путч Гитлера / путч Гитлера и Людендорфа.

Комментарий: В статье говорится, что даже сведения, полученные в мюнхенской пивной за кружкой пива, оказывались полнее и достовернее, чем размытые донесения разведчиков. В мюнхенской пивной в 1923 году произошел Пивной путч, со стороны баварской полиции погибло четыре человека.

Источник: 1) http://fablewar.ru/2011/06/zorge/ 2) http://en.wikipedia.org/wiki/Beer_Hall_Putsch

Автор: Евгений Ярков (Тюмень)

Средняя оценка: 0.78

Комментарии


13. Вопрос 1.6: Когда пронесся слух, что Бруклинскому мосту грозит обрушение, власти Нью-Йорка организовали мероприятие, центральную роль в котором выполняли два десятка ИХ. Какое «имя» носил антагонист одного из НИХ в произведении русского классика?

Ответ: Моська.

Зачёт: точный ответ.

Комментарий: ОНИ – слоны, которые прошли по мосту.

Источник: 1)http://milomalo.ru/?p=15453 2)http://rvb.ru/18vek/krylov/01text/vol3/01fables/085.htm

Автор: Константин Костенко (Гремячинск).

Средняя оценка: 0.76

Комментарии


14. Вопрос 4.23: В 1983 году Стив Джобс занимался проектом по созданию особо мощного ком-пьютера, который он назвал ИКСОМ. Назовите экономический термин, в названии которого есть ИКС.

Ответ: Индекс Биг-Мака.

Комментарий: Компьютеры фирмы Apple называют «маками», а этот, особо мощный, назвали «биг-маком».

Источник:
    1. Уолтер Айзексон «Стив Джобс».
    2. http://ru.wikipedia.org/wiki/Индекс_Биг-Мака

Автор: Ярослав Косарев (Нижний Новгород).

Средняя оценка: 0.74

Комментарии


15. Вопрос 4.28: Героиня одного американского романа не знает имени своего босса, поэтому называет его ИКСОМ. ИКС был у Дженни Кле́менс. Какие слова мы заменили на ИКС?

Ответ: Дядя Сэм.

Комментарий: Героиня романа работает на государственную структуру, при-ближенную к правительству США. Дядя Дженни Клеменс - Марк Твен, настоя-щее имя которого Сэ́мюэл Кле́менс.

Источник:
    1. Кристоф Бакли «Флоренс Аравийская» http://lib.rus.ec/b/132384/read
    2. http://en.wikipedia.org/wiki/Orion_Clemens

Автор: Ярослав Косарев (Нижний Новгород).

Средняя оценка: 0.74

Комментарии


16. Вопрос 2.4: На олимпиаде в Риме в 1960 году судьи не сразу обнаружили, что спортсмен из Туниса выступал на командных соревнованиях один, каждый раз выходя под другими именем. В том же виде состязаний в 1924 году для разрешения спорных судейских решений состоялись две настоящие ОНИ. Назовите ИХ.

Ответ: Дуэли

Зачёт: По слову “Дуэль”.

Комментарий: Из-за маски лица спортсмена не видно и судьи не сразу догадались, что выступает один и тот же спортсмен. В 1924 году споры были настолько ожесточенные, что некоторые решили выяснить отношения в более справедливом виде.

Источник: интересные-факты.com/99-ljubopytnyh-olimpijskih-istorij/

Автор: Евгений Ярков (Тюмень)

Средняя оценка: 0.73

Комментарии


17. Вопрос 3.20: Англичанин Диккенс замечает, что во всех порядочных кровавых мелодрамах трагические сцены обязательно должны перемежаться с комическими. Эти мелодрамы Диккенс сравнивает с НИМ. ЕМУ, вероятно, обязаны фамилией нескольких известных англичан. Назовите ЕГО.

Ответ: бекон.

Зачёт: слоистый бекон, свиная грудинка, грудинка.

Комментарий: «подобно тому, как в свиной грудинке чередуются слои красные и белые». Известны философ Роджер Бэкон, а также философ и художник Фрэнсисы Бэконы. Фамилия Bacon, по одной из основных версий, восходит к бекону.

Источник:
    1. Dickens Ch. Oliver Twist http://www.americanliterature.com/author/charles-dickens/book/oliver-twist/chapter-xvii
    2. https://www.surnamedb.com/Surname/Bacon

Автор: Владислав Декалов (Волгоград)

Средняя оценка: 0.72

Комментарии


18. Вопрос 1.3: В городе Мак-Кинни английское название организации, предоставляющей услуги нанесения татуировок, состоит из двух совпадающих на слух трехбуквенных фрагментов. Как она называется?

Ответ: «Ink Inc».

Зачёт: точный ответ.

Комментарий: «Чернила Инкорпорэйтед».

Источник: http://www.inkincstudio.com/Website/contactus.html

Автор: Александр Пономарев (Пермь).

Средняя оценка: 0.71

Комментарии


19. Вопрос 2.5: Человечеству суждено природой стремиться к новым открытиям. Говоря о подлёдном океане, в котором может существовать жизнь, Сергей Ткаченко задается вопросом о том, кто первым СДЕЛАЕТ ЭТО? Какие четыре слова мы заменили словами “СДЕЛАЕТ ЭТО”?

Ответ: Прорубит окно в Европу.

Зачёт: По словам “окно в Европу” с соответствием формулировки.

Комментарий: На Европе, спутнике Юпитера, существует подлёдный океан, в котором может существовать жизнь, для обнаружения которой достаточно лишь “прорубиться” под лёд. Говоря о заложении Петербурга, Пушкин писал - “Природой здесь нам суждено, в Европу прорубить окно”

Источник: 1) http://shkolazhizni.ru/archive/0/n-17357/ 2) http://ru.wikipedia.org/wiki/В_Европу_прорубить_окно

Автор: Евгений Ярков (Тюмень)

Средняя оценка: 0.68

Комментарии


20. Вопрос 1.11: Персонаж книги Эдуарда Успенского предполагает, что Чебурашка – это ТАКАЯ АЛЬФА. Среди ТАКИХ АЛЬФ есть Апостол, Бортник и Гайдук. Какие два слова, начинающиеся с соседних букв, мы заменили выражением «ТАКАЯ АЛЬФА»?

Ответ: украинская фамилия.

Зачёт: точный ответ.

Источник: 1)http://www.planetaskazok.ru/euspensky/bizneskrokodilagenyuspensky?start=7 2)http://ru.wikipedia.org/wiki/Категория:Украинские_фамилии

Автор: Константин Костенко (Гремячинск).

Средняя оценка: 0.66

Комментарии


21. Вопрос 4.13: На одной карикатуре можно прочитать фразу «сотни стрелков стараются сведать страдания смерти синицы», а также увидеть семь ИХ. Назовите роман, в название которого входят ОНИ.

Ответ: «Пятьдесят оттенков серого».

Зачёт: «50 оттенков серого».

Комментарий: ОНИ - это оттенки серого. На карикатуре изображена своеобразная чёрно-белая радуга.

Источник: http://cs425831.vk.me/v425831584/2fff/_N1kEgcHKOc.jpg

Автор: Сергей Лобачев (Нижний Новгород).

Средняя оценка: 0.65

Комментарии (29)


22. Вопрос 3.4: В Норильском лагере заключённые подняли бунт, требуя относиться к ним как к людям и в первую очередь СДЕЛАТЬ ЭТО. Государственная дума готовит закон, согласно которому в определённых случаях будут не ДЕЛАТЬ ЭТО, а наказывать повышенным штрафом. Какие два слова мы заменили словами СДЕЛАТЬ ЭТО?

Ответ: снять номера.

Зачёт: снимать номера, отобрать/забрать/отбирать/забирать номера.

Комментарий: зэки требовали снять номера с ватников и обращаться к ним по фамилиям. За ряд нарушений, за которые ранее снимали номера (например, езда с тонировкой), собираются просто ввести очень большой штраф.

Источник:
    1. Духон Б. Л., Морозов Г. Н. Братья Старостины http://flibusta.net/b/284385/read
    2. http://www.autoomsk.ru/autonews/show/4879

Автор: Александр Коробейников (Саратов)

Средняя оценка: 0.56

Комментарии


23. Вопрос 5.5: [Указание ведущему: имена собственные прочитать по возможности чётко, но без повторения]
    Флэнн О’БрАйен публиковал романы и рассказы. БрАйан О’НОлан работал чиновником. Майлз НагОпалин вёл сатирическую колонку. Газета «Irish Times» [айриш таймс], задавая вопрос о Флэнне О’БрАйене, использует трёхбуквенное, а не двухбуквенное английское слово. Напишите оба этих слова.

Ответ: are, is.

Зачёт: в любом порядке.

Комментарий: Это всё один и тот же человек, наиболее известный под псевдонимом Флэнн О`Брайен. В газете задавались вопросом: «What are Flann O’Brien?».

Источник: Behan B. Secret Scripture // Irish Times. — 07.30.1960. — P. 6.

Автор: Алексей Бороненко (Челябинск — Москва).

Средняя оценка: 0.56

Комментарии


24. Вопрос 5.32: Внимание, в вопросе есть замены.
    Ирландец СЭмюэль БЕккет записывал свои короткие стихотворения в том числе и на ПЕРВЫХ ВТОРОГО. В название некоторых ВТОРЫХ входит слово со значением «ПЕРВАЯ». Напишите любое из этих названий.

Ответ: [Johnnie Walker] Red Label.

Зачёт: [Johnnie Walker] Black Label; [Johnnie Walker] Blue Label; [Johnnie Walker] Double Black Label; [Johnnie Walker] Green Label; [Johnnie Walker] Gold Label; [Johnnie Walker] Gold Label Reserve; [Johnnie Walker] Platinum Label; [Johnnie Walker] White Label; любое из этих названий по-русски.

Комментарий: ПЕРВЫЕ – этикетки, ВТОРЫЕ – виски.

Источник:
    1. Knowlson J. Damned to Fame. Bloomsbury, London, 1996. PP. 568-9.
    2. http://en.wikipedia.org/wiki/Johnnie_Walker_Black_Label#Blends

Автор: Алексей Бороненко (Челябинск — Москва).

Средняя оценка: 0.54

Комментарии


25. Вопрос 3.11: Поэт ЖозЕ ГОмес ФеррЕйра пишет, что ОНИ взошли на пепле африканских земель. Назовите ИХ одним словом.

Ответ: гвоздИки.

Комментарий: Феррейра - португалец, стихотворение его посвящено перевороту 1974 года, получившему название «революция гвоздик». Один из факторов успеха этого переворота - потеря влияния Португалии в колониях в Африке и последующее их отделение от метрополии путем партизанских войн и восстаний.

Источник:
    1. Из современной португальской поэзии. - Издательство "Прогресс", 1980 - с. 68.
    2. http://ru.wikipedia.org/wiki/Революция_гвоздик

Автор: Владислав Декалов (Волгоград)

Средняя оценка: 0.54

Комментарии


26. Вопрос 3.32: Во второй половине XX века был введён запрет на использование яркого искусственного света рядом с озером ТекАпо. В 1965 году у озера была построена ОНА. Какая ОНА располагается приблизительно на 51 градусе 28 минутах северной широты?

Ответ: Гринвичская.

Зачёт: Гринвичская обсерватория, Королевская Гринвичская обсерватория.

Комментарий: над Текапо почти всегда чистое небо, а искусственный свет был запрещён, чтобы не мешать работе приборов обсерватории. Долготу Гринвичской обсерватории мы приводить не стали.

Источник:
    1. http://re-actor.net/space/7746-tecapos-sky.html
    2. http://ru.yktravelphoto.com/places/ozero-tekapo/2087
    3. http://ru.wikipedia.org/wiki/Гринвичская_обсерватория

Автор: Артём Викторов (Саратов)

Средняя оценка: 0.53

Комментарии


27. Вопрос 1.12: Когда вюртембергский герцог Карл Евгений во главе многочисленной свиты собрался на венецианский карнавал, ОНИ не выпустили правителя из города. Статья 562 Гражданского кодекса Российской Федерации рассказывает об ИХ правах при продаже предприятия. Назовите ИХ.

Ответ: кредиторы.

Зачёт: точный ответ.

Источник: 1)http://books.google.ru/books?hl=ru&id=JmVcAAAAMAAJ&q=карнавал#search_anchor 2)http://www.gk-rf.ru/statia562 3) Автор: Александр Пономарев (Пермь).

Средняя оценка: 0.51

Комментарии


28. Вопрос 3.2: Одна статья назвала подверженного травмам центрового Грега Одена ИКСОМ. Какие четыре слова мы заменили ИКСОМ?

Ответ: колосс на глиняных ногах.

Зачёт: точный ответ.

Комментарий: центровые обычно довольно высокого роста, но подверженность травмам делает положение этого конкретного центрового довольно шатким.

Источник: http://www.championat.com/basketball/article-172472-komanda-vsekh-zvjozd-lazareta.html

Автор: Артём Викторов (Саратов)

Средняя оценка: 0.51

Комментарии


29. Вопрос 2.9: В одном произведении офицер, отмечая военные успехи, приближающие разгром противника, сравнивает карту с НЕЙ. Для определенных целей над ней произносят молитву святого Лазаря. Назовите ЕЁ двумя словами.

Ответ: Кукла Вуду

Зачёт: Точный ответ

Комментарий: Склонившись над картой, офицер втыкал иголочки, отмечая передвижения войск. Кукла Вуду, как правило, используется для лечения, обретения удачи, здоровья. А вот использование для “мести”, по большей части, заслуга Голливуда.

Источник: 1) http://www.snob.ru/profile/7986/blog/58790 2) http://proza.ru/2013/10/16/567

Автор: Евгений Ярков (Тюмень)

Средняя оценка: 0.51

Комментарии


30. Вопрос 4.22: Собор Парижской Богоматери строился более 180 лет, и за время его строи-тельства господствовавший в архитектуре стиль несколько раз менялся. От-ветьте одним словом, с кем в связи с этим сравнил Собор Парижской Богомате-ри Виктор Гюго?

Ответ: Химера.

Комментарий: Собор Парижской Богоматери является постройкой смешанного стиля. «Эта главная церковь, церковь-прародительница, является среди древ-них церквей Па-рижа чем-то вроде химеры: у неё голова одной церкви, конеч-ности другой, торс третьей и что-то общее со всеми». Любопытно, что химеры на соборе появились во время его ре-ставрации, уже после выхода романа.

Источник:
    1. http://lib.ru/INOOLD/GUGO/sobor.txt
    2. http://ru.wikipedia.org/wiki/Собор_Парижской_Богоматери

Автор: Николай Слюняев (Нижний Новгород).

Средняя оценка: 0.49

Комментарии (1)


31. Вопрос 2.19: Возлюбленная Россетти, Джейн Бёрден, вышла замуж за другого художника. Брак с этим художником впоследствии оказался для Джейн адом, и она стала регулярно уходить от мужа, чтобы подолгу проводить время с Россетти. В образе кого изображена Джейн на картине Россетти 1874 года?

Ответ: Персефоны

Зачёт: Прозерпины

Комментарий: Как и античная богиня, Джейн часть времени проводила с возлюбленным, а часть в аду с Уильямом Моррисом.

Источник: 1) http://ru.wikipedia.org/wiki/Прозерпина_(картина) 2) http://ru.wikipedia.org/wiki/Прозерпина 3) http://en.wikipedia.org/wiki/Jane_Morris 4) http://ru.wikipedia.org/wiki/Моррис,_Уильям

Автор: Евгений Ярков (Тюмень)

Средняя оценка: 0.46

Комментарии


32. Вопрос 1.5: Сайт sporter.md [спортер точка эм дэ] сообщает, что при использовании ТУШКАНА важно регулярно менять сторону вдоха. Что автор Вопроса заменил ТУШКАНОМ?

Ответ: кроль.

Зачёт: точный ответ.

Комментарий: тушканчик -> тушкан, кролик -> кроль.

Источник: http://sporter.md/posts/?tag=кроль

Автор: Александр Пономарев (Пермь).

Средняя оценка: 0.45

Комментарии


33. Вопрос 4.24: В 1982 году Стив Джобс очень рассчитывал на то, что он станет обладателем почётного звания, но в итоге, можно сказать, СДЕЛАЛ ЭТО. По некоторым дан-ным Вишваната́н А́нанд впервые СДЕЛАЛ ЭТО в 1992 году. Какие два слова мы заменили словами СДЕ-ЛАЛ ЭТО?

Ответ: Проиграл компьютеру.

Комментарий: Как известно, в 1982 году звание «человек года по версии жур-нала «Time» получил персональный компьютер.

Источник:
    1. Уолтер Айзексон «Стив Джобс».
    2. http://www.chessgames.com/perl/chess.pl?pid=15948&pid2=12088
    3. http://uzweb.uz.ac.zw/science/maths/zimaths/chess.htm

Автор: Ярослав Косарев (Нижний Новгород).

Средняя оценка: 0.45

Комментарии


34. Вопрос 1.26: В мультфильме «Баба-Яга против» злодеи решили совершить кражу, используя этот предмет. При этом Змей Горыныч сразу был отвергнут при выборе исполнителя преступления. Что это за предмет?

Ответ: шапка-невидимка.

Зачёт: точный ответ.

Источник: мультфильм «Баба-Яга против», выпуск 2.

Автор: Александр Пономарев (Пермь).

Средняя оценка: 0.45

Комментарии


35. Вопрос 3.25: Журналист сравнил главный купол Тадж-МахАла с мраморным ИМ. ОН присутствует в названии известного романа, в первой главе которого говорится о заказе, поступившем на лионские ткацкие фабрики. Назовите ЕГО двумя словами.

Ответ: воздушный шар.

Зачёт: точный ответ.

Комментарий: купол, кажется, готов улететь в небо. В романе «Пять недель на воздушном шаре» в первой главе лионские фабрики получили заказ на производство шёлковой тафты для оболочки воздушного шара.

Источник:
    1. GEO Traveller. 2004. №11.
    2. Верн Ж. Пять недель на воздушном шаре http://lib.ru/INOFANT/VERN/balloon.txt

Автор: Артём Викторов (Саратов)

Средняя оценка: 0.45

Комментарии


36. Вопрос 5.17: Ребёнок из фильма ХичкОка удивлён и расстроен тем, что ЕГО удалось подстрелить. И говорит, что ЕГО ещё нужно было накормить клевером и даже подковать. Назовите ЕГО.

Ответ: кролик.

Зачёт: заяц.

Комментарий: у кролика с собой всегда четыре талисмана, приносящих удачу, — собственные лапки. Но это ему не помогло. Ребёнок предположил, что если бы кролика подковали и накормили клевером, он бы не умер.

Источник: Неприятности с Гарри (The Trouble with Harry), реж. А. Хичкок, 1955 г. — 35-37 минуты.

Автор: Сергей Спешков.

Средняя оценка: 0.43

Комментарии


37. Вопрос 4.12: Размышляя о том, что гены не всегда копируются безошибочно, Ри́чард До́кинз упомянул «бабушкин шёпот». А как этот «бабушкин шёпот» называем мы?

Ответ: Испорченный телефон.

Зачёт: Сломанный телефон.

Источник:
    1. http://warrax.net/93/15/blind_watchmaker.pdf
    2. http://terebess.hu/keletkultinfo/The_Blind_Watchmaker.pdf

Автор: Сергей Лобачев (Нижний Новгород).

Средняя оценка: 0.42

Комментарии


38. Вопрос 5.6: На гравюре Дэвида МАртина ОНИ аккуратно разложены на столе. На картине РЕмбрандта, описывающей более позднее событие, ОНИ разбросаны по полу. Назовите ИХ.

Ответ: тридцать сребреников.

Зачёт: 30 сребреников.

Комментарий: на гравюре Мартина запечатлён момент, когда с Иудой рассчитываются за будущее предательство. На картине Рембрандта раскаявшийся Иуда возвращает монеты. И в том, и в другом случае художники скрупулёзно изобразили все тридцать монет.

Источник:
    1. Martin D. Judas Agrees to Betray Jesus (http://goo.gl/dylmgd).
    2. http://en.wikipedia.org/wiki/Thirty_pieces_of_silver

Автор: Сергей Спешков.

Средняя оценка: 0.42

Комментарии


39. Вопрос 1.34: Вэл Льюис пишет, что впервые ЯКОРЯ появились ещё в древнем Египте, причем особо ценными были ЯКОРЯ с увеличенным количеством пальцев. Какие два слова, начинающиеся на одну букву, мы заменили на «ЯКОРЯ»?

Ответ: корабельные кошки.

Зачёт: корабельные коты.

Источник: 1)http://olt-z-s.livejournal.com/45487.html 2)http://en.wikipedia.org/wiki/Ship's_cat

Автор: Константин Костенко (Гремячинск).

Средняя оценка: 0.41

Комментарии


40. Вопрос 4.7: Ибн-Батту́та, посетивший в 14 веке Константинополь, отказался совершать некое дей-ствие и, в результате, не смог попасть в собор святой Софии. Назовите должность, название которой происходит от этого действия.

Ответ: Целовальник.

Комментарий: Ибн-Баттута был мусульманином и отказался присягать на веру, целуя крест. Перед вступлением в должность целовальники клялись служить честно и в под-тверждение своей клятвы целовали крест.

Источник:
    1. http://ru.wikipedia.org/wiki/Ибн_Баттута
    2. http://flibusta.net/b/177142/read
    3. http://ru.wikipedia.org/wiki/Целовальник

Автор: Сергей Фокин (Санкт-Петербург).

Средняя оценка: 0.4

Комментарии


41. Вопрос 5.33: Третьего из близнецов РЕвковых, родившихся более полувека назад, назвали Джоном. Назовите имена первого и второго братьев.

Ответ: Юрий, Герман.

Зачёт: в любом порядке.

Комментарий: тройняшек, родившихся в 1962 году, по тогдашней моде назвали в честь космонавтов. В данном случае в честь первых людей, осуществивших орбитальный космический полёт: Гагарина, Титова и Гленна.

Источник: Харитонова Е.В. От «Востока» до МКС // Московский журнал. — №4, 2011 (http://goo.gl/kLAqfQ).

Автор: Сергей Спешков

Средняя оценка: 0.4

Комментарии


42. Вопрос 5.34: У персонажа Джеймса Джойса ОНА выглядит злой. Но после несложных манипуляций ОНА уже хохочет над моряцкой байкой. Назовите ЕЁ.

Ответ: татуировка.

Зачёт: тату.

Комментарий: у персонажа на груди вытатуирован хмурый профиль товарища. Персонаж растягивает пальцами кожу, заставляя профиль «хохотать».

Источник: Джойс Дж. Улисс (http://www.james-joyce.ru/ulysses/ulysses-text.htm).

Автор: Сергей Спешков.

Средняя оценка: 0.4

Комментарии


43. Вопрос 1.30: Инженер НАСА Джеймс Оберг выступает против наличия ЭТОГО в космосе, объясняя свою точку зрения психическим перенапряжением. Россиянин Юрий Гидзенко – наоборот, высказывается за наличие ЭТОГО и напоминает, что порой космонавты ждут спасателей несколько суток. Третий зал какого московского музея в основном содержит ЭТО XV–XIX веков?

Ответ: Оружейная палата.

Зачёт: точный ответ.

Комментарий: ЭТО – оружие. Например для защиты космонавтов от хищных зверей, добывания пищи охотой, подачи сигналов после приземления. Оберг боится, что на борту станции все друг друга перестреляют.

Источник: 1)http://news.bbc.co.uk/hi/russian/russia/newsid_7250000/7250461.stm http://ru.wikipedia.org/wiki/Оружейная_палата.

Автор: Александр Пономарёв (Пермь).

Средняя оценка: 0.39

Комментарии


44. Вопрос 5.18: Дмитрий Чернышёв рассказывает, что итальянские художники иногда рисовали на своих полотнах пауков. Они не хотели допустить появления того, что описывается в начале известного литературного произведения. Напишите фамилию его автора.

Ответ: Гашек.

Зачёт: точный ответ.

Комментарий: пауков изображали, чтобы мухи не засиживали картины. Засиженный мухами портрет Франца-Иосифа описывается в начале «Похождений бравого солдата Швейка».

Источник:
    1. Чернышёв Д.А. Как люди думают. — М.: Манн, Иванов и Фербер, 2013. — с. 149.
    2. Гашек Я. Похождения бравого солдата Швейка (http://lib.ru/GASHEK/shveik1.txt).

Автор: Алексей Бороненко (Челябинск — Москва).

Средняя оценка: 0.38

Комментарии (1)


45. Вопрос 4.5: Ответьте двумя словами, что Антуан де Сент-Экзюпери в своей автобиографи-ческой повести сравнил с подвенечной фатой.

Ответ: Конденсационный след.

Зачёт: Инверсионный след; след самолёта; самолётный след; реактивный след.

Источник:
    1. http://flibusta.net/b/79277/read
    2. http://ta-vi-ka.blogspot.com/2013/05/samolet.html

Автор: Сергей Лобачев (Нижний Новгород).

Средняя оценка: 0.37

Комментарии


46. Вопрос 2.17: Герой Алессандро Барикко вспоминает, как бедняки, отправлявшиеся в Америку практически в лохмотьях, за двадцать дней путешествия оставляли лайнер без НИХ. В музее замка Колдиц, который в разное время был трудовой колонией, больницей для умалишенных и концлагерем, можно обнаружить лопаты из ножек кроватей, пилы из обломков бритвенных лезвий и связки ИХ. Назовите ИХ.

Ответ: Простыни

Зачёт: Наволочки

Комментарий: Бедняки шили из выдаваемого им постельного белья белоснежную новую одежду. В музее содержатся самодельные инструменты для побега, в том числе лестницы из простыней.

Источник: 1) http://lib.ru/INPROZ/BARIKKO/novecento.txt 2) http://ru.wikipedia.org/wiki/Замок_Колдиц

Автор: Евгений Ярков (Тюмень)

Средняя оценка: 0.36

Комментарии


47. Вопрос 1.28: Из этого «животного» можно сделать, например, птичку, жирафа, кошку, собаку, петуха, лягушку, тюленя, лебедя, черепашку и даже волка. Какой европеец создал это «животное»?

Ответ: Эрнё Рубик.

Зачёт: Рубик.

Комментарий: «животное» - змейка. Все это можно из нее собрать.

Источник: 1)http://www.igrushka24.ru/catalog/zmeyka-bolshaya-rubiks-twist-24-elementa/ 2)http://ru.wikipedia.org/wiki/Змейка_Рубика

Автор: Александр Пономарев (Пермь).

Средняя оценка: 0.36

Комментарии (1)


48. Вопрос 1.36: При ЕЁ использовании запрещается пользоваться клеем. Иногда ЕЁ роль исполняет металлический наконечник пожарного рукава. В длительных мероприятиях вместо НЕЁ могут использовать ленту. Назовите ЕЁ.

Ответ: эстафетная палочка.

Зачёт: точный ответ.

Комментарий: лента используется, например, в экидене – эстафете на марафонской дистанции.

Источник: 1)http://ru.wikipedia.org/wiki/Эстафета 2)http://www.severnyemn.ru/gazety/2009-08_32/pozharniki.htm 3)http://www.ufsin.omsk.ru/about/info/news/620/ 4)http://ru.wikipedia.org/wiki/Пожарный_ствол 5)http://ru.wikipedia.org/wiki/Экиден

Автор: Александр Пономарев (Пермь).

Средняя оценка: 0.35

Комментарии


49. Вопрос 3.26: Назовите имя того, кого польский поэт ЮлиАн ТувИм назвал разгульным лесовиком.

Ответ: Пан.

Зачёт: точный ответ.

Комментарий: в Польше вообще много панов, но в данном случае это греческий бог.

Источник: http://lall.livejournal.com/93834.html

Автор: Александр Коробейников (Саратов)

Средняя оценка: 0.31

Комментарии


50. Вопрос 2.24: Перед Вами часть постера фильма о НЁМ. Персонаж известного произведения сожалеет, что преступник будет вскоре пойман, ведь ему хочется посмотреть, как тот выкрутится, дойдя до НЕГО. Назовите это произведение.

Ответ: Убийства по алфавиту

Зачёт: The A.B.C. murders

Комментарий: Сюжет этого детектива Агаты Кристи строится на убийствах в алфавитном порядке. Сначала убивают старушку Ашер в Андовере, затем Бетти в Бексхилле и так далее. На букву X (икс) в английском языке не так много слов, а уж населенных пунктов в Англии и соответствующих фамилий практически нет. Малькольм Икс, чернокожий борец с расизмом, на постере его изображение обрамлено соответствующей буквой. Икс - 24-я буква английского алфавита.

Источник: 1) http://www.loveread.ec/read_book.php?id=2653&p=17 2) http://www.peoples.ru/state/statesmen/malcolm_x/ 3) http://www.kinopoisk.ru/film/1105/

Автор: Евгений Ярков (Тюмень)

Средняя оценка: 0.3

Комментарии


51. Вопрос 1.29: Одна иностранка учила русский язык. Чтобы справиться с одной буквой, ей посоветовали использовать, так сказать, фрагмент слова «table» [тэйбл]. Что это за буква?

Ответ: Ы.

Зачёт: точный ответ.

Комментарий: посоветовали в слове «table» вычленить звук между «b» и «l»,

Источник: http://esquire.ru/lost-in-translation

Автор: Константин Костенко (Гремячинск).

Средняя оценка: 0.29

Комментарии


52. Вопрос 2.13: Привратниковый отдел ИХ желудка снабжен мощной мускулатурой. На одном изображении ОНИ устраивают пикник на природе, во время которого сидят перед положенным на траву тортом и ждут. Назовите ИХ.

Ответ: Муравьеды

Зачёт: Точный ответ

Комментарий: Торт служит приманкой для муравьев, которыми и планируют полакомиться муравьеды. У муравьедов зубов нет, поэтому жесткую пищу они перетирают мышцами желудка.

Источник: 1) http://ru.wikipedia.org/wiki/Муравьед 2) http://std3.ru/f0/34/1380730165-f0348f12612dedb735bb0626fed1fee1.jpg

Автор: Евгений Ярков (Тюмень)

Средняя оценка: 0.29

Комментарии


53. Вопрос 5.16: Одно сражение Корейской войны происходило в узкой долине. Американцы несколько раз атаковали противников, пока те, наконец, не были опрокинуты. В неофициальном названии места сражения упоминается ОН. Для ЕГО комнатной разновидности предлагают использовать набор солдатиков. Назовите ЕГО.

Ответ: боулинг.

Зачёт: кегельбан.

Комментарий: узкая долина, в конце которой находились противники, напомнила американцам дорожку для боулинга. Простейший вариант боулинга для детей можно устроить с помощью солдатиков и подходящего мячика.

Источник:
    1. http://en.wikipedia.org/wiki/Battle_of_the_Bowling_Alley
    2. Army Man Bowling // Instructables (http://goo.gl/QD19jt).

Автор: Сергей Спешков.

Средняя оценка: 0.28

Комментарии


54. Вопрос 1.27: По данным ихтиолога Джорджа Берджесса, в 2008 году число нападений акул на людей резко сократилось по сравнению с предыдущим годом. Ученый полагает, что снижение числа нападений произошло в связи с НИМ. Назовите ЕГО одним словом.

Ответ: кризис.

Зачёт: синонимичные ответы.

Комментарий: который мировой финансовый. По мнению Берджесса, он вынудил людей больше работать и меньше заниматься водными видами спорта.

Источник: http://news.mail.ru/incident/2383220/

Автор: Александр Пономарев (Пермь).

Средняя оценка: 0.28

Комментарии


55. Вопрос 3.17: ОН открывается в самом конце первой части кинотрилогии. ОН дал название самолёту-беспилотнику, который позволяет исследователям снимать задымлённые области возле действующих вулканов. Назовите ЕГО двумя словами, начинающимися на соседние буквы алфавита.

Ответ: глаз дракона.

Зачёт: драконий глаз.

Комментарий: беспилотные аппараты “Dragon Eye” становятся “глазами” исследователей, так как могут собирать данные, которые трудно получить без специальных средств. Ну а кому, как не дракону, хорошо ориентироваться в условиях сверхвысоких температур и практически нулевой видимости? В конце первой части кинотрилогии про Хоббита компания главных героев может увидеть Одинокую гору, где и обитает дракон Смауг.

Источник:
    1. Популярная механика. 2013. №10. С. 92.
    2. http://en.wikipedia.org/wiki/The_Hobbit:_An_Unexpected_Journey

Автор: Владислав Декалов (Волгоград)

Средняя оценка: 0.27

Комментарии


56. Вопрос 2.14: Внимание, в вопросе есть замены.
    Юная героиня рассказа Хайнлайна “Клифф и калории” сожалеет, что растет не ИКС, а ИГРЕК. Произведение 1988 года, которое можно читать как ИКС, так и ИГРЕК, в конце содержит ответы. Что мы заменили на ИКС и на ИГРЕК?

Ответ: По горизонтали, По вертикали

Зачёт: В любом порядке

Комментарий: Героиня сожалеет, что набирает вес, притом, что её рост остается прежним. “Пейзаж, написанный чаем” Милорада Павича – это роман-кроссворд, ответы на который написаны в конце книги “вверх тормашками”. ИКС и ИГРЕК – небольшие намёки на абсциссу и ординату.

Источник: 1) http://lib.ru/INPROZ/PAWICH/endofnovelrus.txt 2) Р. Хайнлайн, “Клифф и калории”. 3) http://lib.ru/INPROZ/PAWICH/tea.txt

Автор: Евгений Ярков (Тюмень)

Средняя оценка: 0.26

Комментарии


57. Вопрос 2.23: Рассказывая о девушке из французских легенд тринадцатого века, литературовед Амфитеатров отмечает, что отец её был дворянином и разбойником. “Впрочем, - продолжает Амфитеатров, - по тому грозному времени - это ОН”. В одном из слов цитаты мы пропустили шесть букв. Напишите эти шесть алфавитных букв.

Ответ: П,Л,Е,А,З,М

Зачёт: В любом порядке

Комментарий: Дворянин и разбойник по тому времени - это плеоназм, то есть - оборот речи, в котором происходит дублирование некоторого элемента смысла. Это уже намного позднее дворянина-разбойника можно было считать оксюмороном. Словосочетание «алфавитные буквы» тоже своего рода плеоназм.

Источник: 1) http://ru.wikipedia.org/wiki/Плеоназм 2) http://az.lib.ru/a/amfiteatrow_a_w/text_1901_teny_oldorfo.shtml

Автор: Евгений Ярков (Тюмень)

Средняя оценка: 0.25

Комментарии


58. Вопрос 1.13: На одном из рисунков Артемия Лебедева слева находится изображение съедобного растения, а справа – такое же изображение, только выполненное в оттенках серого цвета. Что это за растение?

Ответ: цветная капуста.

Зачёт: точный ответ.

Комментарий: справа – «черно-белая капуста».

Источник: http://tema.ru/crea-gif/kapusta.html.

Автор: Александр Пономарев (Пермь).

Средняя оценка: 0.25

Комментарии (1)


59. Вопрос 2.8: С каждым днем времена меняются. Одна разновидность часов была скопирована в Уппсале и помещена в одном российском селе, однако точное время не показывала. Семь часов утра на таких часах можно назвать ИМ. Назовите ЕГО двумя словами.

Ответ: Время колокольчиков

Зачёт: Точный ответ

Комментарий: Речь идет о цветочных часах, посаженных Карлом Линнеем и перенесенных в село Михайловское. Из-за разницы в климате и природной среде цветы раскрывались не так, как в Уппсале. Утром раскрываются колокольчики, что может немного походить на своеобразный будильник. Время колокольчиков - известное произведение Александра Башлачева, а слова “с каждным днем времена меняются” - цитата из этого произведения.

Источник: 1) http://lit.1september.ru/article.php?ID=200200605 2) http://herbalogya.ru/textbook/flower_clock.php

Автор: Евгений Ярков (Тюмень)

Средняя оценка: 0.23

Комментарии


60. Вопрос 2.10: ОНА была изображена на лицевой стороне купюры, с обратной стороны которой находилась погОня. В одной сказке ОНА приходит к сестрам, после чего одна сестра, обернувшись тазами, ползает черепахой, другая - начинает ткать по-паучьи, а третья становится золотой пчёлкой. Назовите ЕЁ.

Ответ: Белочка

Зачёт: Белка

Комментарий: На банкноте в 50 белорусских копеек была изображена белочка, а с обратной стороны - Погоня - герб независимой Белоруссии 1991-1995 годов. Когда приходит белочка, становится довольно интересно.

Источник: 1) http://hobbitaniya.ru/tatar/tatar7.php 2) http://ru.wikipedia.org/wiki/Белорусский_рубль 3) http://ru.wikipedia.org/wiki/Погоня_(герб)

Автор: Евгений Ярков (Тюмень)

Средняя оценка: 0.21

Комментарии


61. Вопрос 4.15: Мераб Мамардашви́ли не оставил после себя письменного наследия и был буквально сжит со свету в разгар тбилисской демократии. Поэтому во многих источниках Мераба ПРОПУСК. Какие три слова, начинающиеся на одну и ту же букву, мы пропустили в этом вопросе?

Ответ: Сравнивают с Сократом.

Комментарий: Мамардашвили - советский философ. Сократ тоже не оставил после себя письменных произведений и пострадал в том числе за свои высказывания об афинской демократии.

Источник:
    1. http://newshevelev.narod.ru/hronograph/godpogodi/yanvar/second/2.htm
    2. http://ru.wikipedia.org/wiki/Мамардашвили,_Мераб_Константинович

Автор: Ярослав Косарев (Нижний Новгород).

Средняя оценка: 0.2

Комментарии


62. Вопрос 4.16: Донато Джанотти, считавшийся приверженцем республиканских убеждений, заказал Ми-кела́нджело Буонарро́ти ЕГО. Назовите ЕГО двумя словами, начинающимися на одну и ту же букву.

Ответ: Бюст Брута.

Комментарий: Как известно, Брут играл одну из главных ролей в заговоре против Цезаря, который, по мнению заговорщиков, предал идеи республики, став единоличным правите-лем.

Источник: http://ru.wikipedia.org/wiki/Брут_(Микеланджело)

Автор: Ярослав Косарев (Нижний Новгород).

Средняя оценка: 0.2

Комментарии


63. Вопрос 2.7: Перед Вами ОНИ, установленные в Раифском монастыре. Их может дополнить ноктурлабиум. Назовите ИХ двумя словами.

Ответ: Солнечные часы

Зачёт: Точный ответ

Комментарий: Солнечный свет проникает сквозь отверстия и падает на соответствующие шкалы на постаменте. Ноктурлабиум - это часы для ночи, ведь солнечные часы ночью не работают.

Источник: 1) http://www.panoramio.com/photo/90861490 2) http://ru.wikipedia.org/wiki/Ноктурлабиум

Автор: Евгений Ярков (Тюмень)

Средняя оценка: 0.16

Комментарии


64. Вопрос 1.1: Остроумному Михаилу Жванецкому приписывают фразу о том, что жизнь подобна роялю: клавиша белая, клавиша черная... А какая часть рояля упоминается в самом конце этой фразы?

Ответ: крышка.

Зачёт: точный ответ.

Источник: http://addfun.ru/2009/06/10/luchshie-aforizmy-zhvaneckogo.html.

Автор: Александр Пономарев (Пермь).

Средняя оценка: 0.12

Комментарии


65. Вопрос 3.21: Терпящий поражение герой Гёте образно замечает, что вОроны, несущие с поля боя плохие новости, ДЕЛАЮТ ЭТО. Кто ДЕЛАЛ ЭТО по забывчивости?

Ответ: Тесей.

Зачёт: Тезей.

Комментарий: ДЕЛАТЬ ЭТО – плыть под чёрными парусами.

Источник:
    1. Гёте И. В. Фауст http://lib.ru/POEZIQ/GETE/faust.txt
    2. http://ru.wikipedia.org/wiki/Тесей

Автор: Владислав Декалов (Волгоград)

Средняя оценка: 0.12

Комментарии


66. Вопрос 1.2: В одном из фильмов о Полицейской академии видно, что у одного из персонажей на НИХ слева звёзды, а справа полосы. Специалист по имиджу Алексей Алексеев отмечает, что ОНИ стройнят, и рекомендует использовать ИХ. Вместо чего?

Ответ: ремня.

Зачёт: точный ответ.

Комментарий: ОНИ - подтяжки.

Источник: 1)Х/ф «Полицейская академия-7. Миссия в Москве» 2)http://www.vedomosti.ru/friday/article/2007/12/28/11380

Автор: Константин Костенко (Гремячинск).

Средняя оценка: 0.12

Комментарии (2)


67. Вопрос 4.30: В 1977 году в ГДР ЕЁ перестали публиковать по решению властей. Согласно некоторым данным, на известном объекте ОНА пополняется в среднем раз в две недели. Назовите ЕЁ.

Ответ: Статистика самоубийств.

Комментарий: Власти скрывали статистику самоубийств, иначе стал бы очеви-ден низкий уровень жизни в стране. Упомянутый в вопросе объект - мост «Зо-лотые ворота».

Источник:
    1. Фильм «Жизнь других» реж. Флориан Хенкель фон Доннерсмарк.
    2. http://lossofsoul.com/DEATH/suicide/statistic.htm
    3. http://foturist-ru.livejournal.com/1064.html

Автор: Ярослав Косарев (Нижний Новгород).

Средняя оценка: 0.1

Комментарии


68. Вопрос 5.26: [Указание ведущему: чётко прочитать в последнем предложении окончание слов ПЕРВЫЕ и ВТОРЫХ]
    Персонаж МатЕя ВИшнека разглядывает свою одежду и говорит, что, в общем-то, мы все носим на себе ПЕРВЫЕ, которые как бы живут на нас, как ВТОРЫЕ. Назовите ПЕРВЫЕ и ВТОРЫХ.

Ответ: слова, паразиты.

Зачёт: в любом порядке.

Комментарий: персонаж собственно имеет в виду различные ярлычки и, так сказать, надписи на одежде, которую носит человек.

Источник: Вишнек М. Синдром паники в городе огней. — М.: ООО «Ад Маргинем Пресс», 2012. — с. 222.

Автор: Алексей Бороненко (Челябинск — Москва).

Средняя оценка: 0.1

Комментарии (2)


69. Вопрос 3.34: Героиня Карла ХайАсена обручилась с парнем, но тот слишком злоупотреблял ЭТИМ, и она расторгла помолвку, а обручальное кольцо на всякий случай не вернула. Назовите ЭТО одним английским словом.

Ответ: пирсинг.

Зачёт: piercing.

Комментарий: персонаж стал маньяком пирсинга, и обручальное кольцо героиня ему не отдала, чтобы тот его тоже куда-нибудь себе не вставил.

Источник: Хайасен К. Хворый пёс http://flibusta.net/b/161137/read

Автор: Александр Коробейников (Саратов)

Средняя оценка: 0.1

Комментарии


70. Вопрос 3.33: В произведении «Грязные руки» один персонаж вручает другому грязные руки и говорит, что это очень удобная вещь, полезная летом и избавляющая от многих хлопот. Какие слова мы заменили словами «грязные руки»?

Ответ: холодное сердце.

Зачёт: точный ответ, возможно в кавычках.

Комментарий: речь идёт о героях сказки Гауфа “Холодное сердце”. Холодное сердце не бьётся от тревоги, любви или жалости, а летом прекрасно освежает. Замена основана на известном высказывании о чекистах

Источник: Гауф В. Холодное сердце http://www.kostyor.ru/tales/tale63.html

Автор: Александр Коробейников, Артём Викторов (Саратов)

Средняя оценка: 0.09

Комментарии


71. Вопрос 4.6: Джеки Робинсон несмотря ни на что стал в 1947 году игроком Главной бейс-больной лиги. Любопытно, что однажды старший брат Джеки стал вторым. А после КОГО?

Ответ: Джесси Оуэнса.

Зачёт: Оуэнса; Джеймса Оуэнса; Джеймса Кливленда Оуэнса; Джеймса Клив-ленда «Джесси» Оуэнса.

Комментарий: Оба брата были темнокожими и постоянно сталкивались с расо-вой дис-криминацией. Старший брат Мак Робинсон завоевал серебряную олим-пийскую медаль на играх в Берлине, финишировав вторым после другого из-вестного темнокожего атлета - Джесси Оуэнса.

Источник:
    1. http://en.wikipedia.org/wiki/Mack_Robinson_(athlete)
    2. http://ru.wikipedia.org/wiki/Робинсон,_Джеки

Автор: Александр Жданкин (Саратов).

Средняя оценка: 0.08

Комментарии


72. Вопрос 4.14: В произведении Андрея Сенько́ва зрители ДЕЛАЛИ ЭТО на представлении Гудини. Счи-тается, что в неволе дельфины способны покончить жизнь самоубийством, СДЕЛАВ ЭТО. Какие два слова мы заменили словами ДЕЛАТЬ ЭТО?

Ответ: Задерживать дыхание.

Источник:
    1. http://ru.wikipedia.org/wiki/Сен-Сеньков,_Андрей_Валерьевич
    2. http://lib.rus.ec/b/371529/read
    3. http://www.stratege.ru/forums/showthread.php?t=13917

Автор: Сергей Лобачев (Нижний Новгород).

Средняя оценка: 0.08

Комментарии


73. Вопрос 4.34: В одном биографическом сериале главный герой вспоминает о том, как на ка-торге ему приходилось крутить точильное колесо. А где он при этом находится?

Ответ: В казино.

Комментарий: А герой - Федор Достоевский. Точильное колесо ему напомнило колесо ру-летки.

Источник: Сериал «Достоевский» реж. В. Хотиненко, серия №7, седьмая минута.

Автор: Ярослав Косарев (Нижний Новгород).

Средняя оценка: 0.06

Комментарии (1)


74. Вопрос 1.20: В «Махабхарате» Кришна называет ЕГО неодолимым. Когда британский политик Уильям Гладстон поддержал прогрессивный закон, из его слов можно было заключить, что Гладстон заручился ЕГО поддержкой. Назовите ЕГО.

Ответ: время.

Зачёт: точный ответ.

Комментарий: буквально Гладстон сказал: «время на нашей стороне».

Источник: 1)http://ru.wikipedia.org/wiki/Калавада 2)http://dic.academic.ru/dic.nsf/dic_wingwords/488/Время

Автор: Александр Пономарев (Пермь).

Средняя оценка: 0.06

Комментарии


75. Вопрос 2.26: Персонаж одной карикатуры, наклоняясь с зажженной спичкой, спрашивает: “А свои-то куда дел?” и получает ответ из одного слова. Напишите это слово.

Ответ: Стреляли

Зачёт: Точный ответ

Комментарий: Персонаж, по-видимому, товарищ Сухов из фильма “Белое солнце пустыни”. Он встречается с Саидом, который произносит своё знаменитое “Стреляли”, в это раз имея в виду сигареты.

Источник: http://www.anekdot.ru/an/an3309/e330910;2,1.html

Автор: Евгений Ярков (Тюмень)

Средняя оценка: 0.06

Комментарии


76. Вопрос 1.14: Среди организаций, расположенных в оренбургском торговом комплексе «Три мартышки», есть и студия загара, ведущая конкурентную борьбу за клиентов с другими подобными заведениями города. Как эта студия называется?

Ответ: «Место под Солнцем».

Зачёт: точный ответ.

Источник: http://orenburg.spravker.ru/soljarii/studija-zagara-mesto-pod-solncem.htm

Автор: Александр Пономарев (Пермь).

Средняя оценка: 0.06

Комментарии


77. Вопрос 5.4: По одной версии, для ЭТОГО использовали снятую с петель дверь. Так объясняют, почему исполнитель ограничен в пространстве. ЭТО стало набирать популярность после того, как было представлено на Евровидении. В какой стране оно проходило?

Ответ: в Ирландии.

Зачёт: точный ответ.

Комментарий: речь идёт об ирландских степ-танцах. Характерная их особенность — практически неподвижный корпус с плотно прижатыми к бёдрам руками, а также чёткие быстрые движения ногами. Дверь использовали, поскольку это твёрдая ровная поверхность. После успеха на Евровидении танцевальное шоу «Риверданс» стало популярным по всему миру.

Источник:
    1. http://en.wikipedia.org/wiki/Riverdance
    2. http://en.wikipedia.org/wiki/Irish_stepdance

Автор: Сергей Спешков.

Средняя оценка: 0.05

Комментарии


78. Вопрос 3.28: Герой Акунина считает, что на Востоке ПЕРВЫЙ – это что-то вроде вьюнка, который ластится ко всякой крепкой палке, а в России ПЕРВЫЙ прямой и твёрдый, как ВТОРОЕ. Назовите ПЕРВЫЙ и ВТОРОЕ.

Ответ: закон; дышло.

Зачёт: в любом порядке.

Комментарий: конечно, поворачивается, куда надо, но хотя бы имеет свои достоинства.

Источник: Акунин Б. Чёрный город http://flibusta.net/b/303153/read

Автор: Александр Коробейников (Саратов)

Средняя оценка: 0.03

Комментарии


79. Вопрос 1.7: После того, как артист разговорного жанра Михаил Евдокимов стал губернатором Алтайского края, журналист Дмитрий Камышев в статье о нем немного изменил имя литературного персонажа. Что у него получилось?

Ответ: Алтай-Болтай.

Зачёт: точный ответ.

Комментарий: персонаж - Шалтай-Болтай.

Источник: http://www.kommersant.ru/doc/465524.

Автор: Александр Пономарев (Пермь).

Средняя оценка: 0.02

Комментарии


80. Вопрос 2.12: В одном произведении огорчённого ЕГО, одетого в калоши и плащ, утешают, показывая на улыбающихся прохожих и говоря, что именно он – устроитель радостнейшего из сюрпризов. Мария Магдалена в отличие от Марка, Матфея и Луки – не ОН. Назовите ЕГО.

Ответ: Синоптик

Зачёт: Точный ответ

Комментарий: Речь идет о синоптике, ошибочно предсказавшем дождь. Синоптиками называют трех евангелистов – Марка, Матфея и Луку, в евангелиях которых, в отличие от Евангелия от Иоанна, много общего.

Источник: 1) http://az.lib.ru/k/krzhizhanowskij_s_d/text_0270.shtml 2) http://ru.wikipedia.org/wiki/Синоптические_Евангелия

Автор: Евгений Ярков (Тюмень)

Средняя оценка: 0.02

Комментарии


81. Вопрос 4.21: АЛЬФЫ, наряду с приветственными жестами, упоминались в одном московском законе об административной ответственности. Любопытно, что через 2 года после того как этот закон утратил силу, АЛЬФЫ появились на эмблеме Федеральной службы исполнения наказаний. Какое слово мы заменили словом АЛЬФЫ?

Ответ: Фасции.

Комментарий: Упомянутый закон называется «Об административной ответ-ственности за изготовление, распространение и демонстрацию нацисткой символики на территории Москвы». Фасции были символом итальянской фашисткой партии. В римской республике фасции символизировали право магистрата казнить и миловать подданных.

Источник:
    1. http://ru.wikipedia.org/wiki/Фасции
    2. http://www.businesspravo.ru/Docum/DocumShow_DocumID_61882.html
    3. http://kodeks-ob-admin-pravonarusheniyah.com/statya-20-3-propaganda-i-publichnoe-demonstrirovanie-nacistskoj-atributiki-ili-simvoliki/

Автор: Ярослав Косарев(Нижний Новгород).

Средняя оценка: 0.01

Комментарии


82. Вопрос 4.25: Один англоязычный сайт предлагает купить футболку с изображением пяти-угольника, шестиугольника, а также ЕГО контура. По одной из версий ЕГО название этимологически связано со стихийным бедствием. Назовите ЕГО.

Ответ: Орегон.

Комментарий: Надпись на футболке гласит - pentagon, hexagon, oregon. По одной из вер-сий, название штата произошло от французского слова, обозначающего ураган.

Источник:
    1. http://www.lonelydinosaur.com/products/pentagon-hexagon-oregon
    2. http://en.wikipedia.org/wiki/Oregon

Автор: Сергей Лобачев (Нижний Новгород).

Средняя оценка: 0.01

Комментарии


83. Вопрос 3.16: Монахине из одного фильма приходится хранить в памяти множество обрядов чёрной магии. О героине говорят, что она ходячий ИКС. Первый ИКС появился в 1529 году. Какие три слова мы заменили ИКСОМ?

Ответ: индекс запрещённых книг.

Зачёт: список запрещённых книг.

Комментарий: индекс запрещённых книг – список публикаций, которые были запрещены к чтению Римско-католической церковью под угрозой отлучения. В голове героини содержится множество книг по черной магии, которые церковь также не одобряет.

Источник:
    1. http://ru.wikipedia.org/wiki/Индекс_запрещённых_книг
    2. http://ru.wikipedia.org/wiki/To_Aru_Majutsu_no_Index

Автор: Артём Викторов (Саратов)

Средняя оценка: 0.01

Комментарии


84. Вопрос 5.28: Когда ОН пожелал расслабиться после проигрыша крупной суммы денег, ему задали уточняющий вопрос. ОН раздражённо ответил, что ему без разницы. Назовите ЕГО.

Ответ: Бонд.

Зачёт: Джеймс Бонд.

Комментарий: На вопрос о том, взболтать ему водку с мартини в шейкере или просто перемешать, раздражённый Бонд говорит, что ему плевать. Такой вот разрыв шаблона произошёл в фильме «Казино Рояль» 2006 года.

Источник: http://en.wikipedia.org/wiki/Shaken,_not_stirred

Автор: Алексей Бороненко (Челябинск — Москва).

Средняя оценка: 0

Комментарии


85. Вопрос 5.12: Марат КузАев называет ЕЮ пару персонажей криминального сериала, которые могли бы сыграть важную роль в его последних эпизодах. Основные элементы ЕЁ конструкции, как правило, не строго параллельны друг другу. Назовите ЕЁ.

Ответ: двустволка.

Зачёт: точный ответ.

Комментарий: по мнению критика, в сериале есть не просто чеховское ружьё, а целая двустволка. У двуствольных ружей оси стволов обычно расположены под небольшим углом друг к другу. Во-первых, для более точного попадания при стрельбе двумя патронами, во-вторых, для уменьшения последствий отдачи.

Источник:
    1. Кузаев М. Признания опасных людей // GQ-Ревизор. — 27.08.2013 (http://goo.gl/QkEhbd).
    2. http://en.wikipedia.org/wiki/Double-barreled_shotgun

Автор: Алексей Бороненко (Челябинск — Москва).

Средняя оценка: -0.01

Комментарии


86. Вопрос 3.13: Один из подразделов статьи на портале digit.ru [дИджит точка ру] посвящен высоким доходам выпускников IT-специальностей, которые сразу получают зарплаты американского уровня, но не совсем соответствуют требованиям из-за отсутствия опыта. Какое устойчивое выражение стало названием этого подраздела?

Ответ: Молодо, «зелено».

Зачёт: с любой пунктуацией или полным её отсутствием; наличие или отсутствие кавычек тоже не принципиально.

Комментарий: даже молодые айтишники влетают работодателям в копеечку и просят вполне себе «зеленые» зарплаты.

Источник: http://digit.ru/business/20131007/406408414.html

Автор: Владислав Декалов (Волгоград)

Средняя оценка: -0.02

Комментарии


87. Вопрос 2.21: Чтобы запечатлеть притягательное буйство стихии, которое могло привести его к гибели, Уильям Тёрнер попросил матросов сделать с ним… Что?

Ответ: Привязать его к мачте

Зачёт: По смыслу

Комментарий: Как и хитроумный Одиссей, убоявшийся губительного пения сирен, Тёрнер попросил матросов привязать себя к мачте корабля, чтобы видеть игру бушующей стихии.

Источник: http://www.centre.smr.ru/win/pics/pic0108/p0108.htm

Автор: Евгений Ярков (Тюмень)

Средняя оценка: -0.04

Комментарии


88. Вопрос 5.21: Название мультсериала «Looney Tunes» [лУни тьюнс] переводится как «Безумные мелодии», а название мультсериала «Merry Melodies» [мЕрри мЕлодис] — «Весёлые мелодии». СЕсил Адамс рассуждает о разнице между этими параллельно существовавшими мультсериалами и говорит, что в них воплотились божественный дух ПЕРВОГО и божественный дух ВТОРОГО соответственно. Напишите имена ПЕРВОГО и ВТОРОГО.

Ответ: Дионис, Аполлон.

Зачёт: в любом порядке.

Комментарий: Согласно концепции Ницше, изложенной в работе «Рождение трагедии из духа музыки», развитие культуры сводится к борьбе дионисийского и аполлоновского начал — то есть стремления к разрушению, хаосу с одной стороны и стремления к созиданию, гармонии с другой.

Источник:
    1. Adams C. What's the difference between a Looney Tune and a Merrie Melody? // The Straight Dope. — 04.04.1997 (http://goo.gl/XLD6R).
    2. http://ru.wikipedia.org/wiki/Ницше,_Фридрих

Автор: Алексей Бороненко (Челябинск — Москва).

Средняя оценка: -0.04

Комментарии


89. Вопрос 4.4: Согласно одной карикатуре ПЕРВЫЙ и ВТОРОЙ в сумме составляют базу данных ФБР. Назовите ПЕРВЫЙ и ВТОРОЙ.

Ответ: Фейсбук, Инстаграм.

Зачёт: Facebook, Instagram.

Автор: Сергей Лобачев (Нижний Новгород).

Средняя оценка: -0.04

Комментарии (3)


90. Вопрос 4.36: По одной из версий, свое название ОН получил в честь «Общества лучистых», организо-ванного То́машем Заном. Назовите ЕГО.

Ответ: Радий.

Комментарий: Согласно этой версии Мария Склодо́вская-Кюри́ связала назва-ние эле-мента с Польшей. В переводе с латыни radius означает луч.

Источник: http://n-t.ru/ri/ps/pb088.htm

Автор: Сергей Лобачев (Нижний Новгород).

Средняя оценка: -0.04

Комментарии


91. Вопрос 5.13: В 1664 году город перешёл к англичанам. До этого полвека он был столицей провинции. Напишите название этой провинции из двух слов, которые начинаются на одну и ту же букву.

Ответ: Новые Нидерланды.

Зачёт: New Netherland.

Комментарий: Логично, что столицей Новых Нидерландов был Новый Амстердам.

Источник: http://en.wikipedia.org/wiki/New_Netherland

Автор: Алексей Бороненко (Челябинск — Москва).

Средняя оценка: -0.04

Комментарии


92. Вопрос 1.9: В сериале «Остаться в живых» скалы, использованные при съемках одного объекта на острове, на самом деле являются резиновыми декорациями. Именно по этой причине звук на этих съемках пришлось записывать уже в студии. Что это за объект?

Ответ: водопад.

Зачёт: точный ответ.

Источник: http://lost-abc.ru/publ/4-1-2.

Автор: Александр Пономарев (Пермь).

Средняя оценка: -0.05

Комментарии


93. Вопрос 3.8: В 1999 году даже специалисты из МГУ, Ботанического сада и Сельхозакадемии не смогли до конца справиться с поставленной задачей. Назовите основанную в 1954 году международную организацию, которая эту задачу поставила.

Ответ: УЕФА.

Зачёт: UEFA.

Комментарий: нужно было подготовить газон «Лужников» к финалу Кубка УЕФА, но идеально не получилось. Колеблющиеся между ФИФА и УЕФА могли припомнить, что чемпионаты мира начали проводиться несколько раньше 1954 года.

Источник:
    1. http://www.sports.ru/tribuna/blogs/skvortsov/467830.html
    2. http://ru.wikipedia.org/wiki/УЕФА

Автор: Александр Коробейников (Саратов)

Средняя оценка: -0.05

Комментарии


94. Вопрос 2.20: Описывая пожилого Велимира Хлебникова, Давид Бурлюк сравнивал его необычайно выразительные поблекшие глаза с НИМИ. Среди них известны, например, “Горящее судно”, “Закат в парке Петуорта” и “Восход с морскими чудовищами”. Назовите ИХ.

Ответ: Картины Уильяма Тёрнера

Зачёт: Пейзажи Тёрнера, в целом по смыслу и упоминанию Тёрнера

Комментарий: Картинам Тёрнера свойственна сильная размытость, отсутствие четких очертаний, неясность и притом, довольно яркие и выразительные тона.

Источник: 1) http://az.lib.ru/l/liwshic_b_k/text_0080.shtml 2) http://ru.wikipedia.org/wiki/Список_картин_Уильяма_Тёрнера 3) http://smallbay.ru/article/art_england_nineteenth_century.html

Автор: Евгений Ярков (Тюмень)

Средняя оценка: -0.05

Комментарии


95. Вопрос 5.36: Георгиевский крест — высшая гражданская награда Великобритании — был учреждён в 1940 году как аналог военного креста Виктории. Один из элементов оформления креста Виктории присутствует и на Георгиевском кресте, но с небольшим добавлением. Напишите оба эти элемента.

Ответ: V, VI.

Зачёт: точный ответ.

Комментарий: в первом случае это инициал имени «Виктория», во втором — порядковый номер короля Георга Шестого. Редакторская группа пятого этапа ОВСЧ благодарит всех участников за игру и передаёт слово редакторам шестого этапа.

Источник:
    1. http://en.wikipedia.org/wiki/George_Cross
    2. http://en.wikipedia.org/wiki/Victoria_Cross

Автор: Сергей Спешков.

Средняя оценка: -0.06

Комментарии


96. Вопрос 1.23: В одном из фильмов про Риддика заглавный герой оказывается в невесомости в одном из отсеков корабля и, чтобы создать завесу, использует устройство. Это устройство после небольшой переделки используют граффитисты. Что это за устройство?

Ответ: огнетушитель.

Зачёт: точный ответ.

Источник: 1)http://ru.wikipedia.org/wiki/Хроники_Риддика:_Тёмная_ярость 2)http://graffitimarket.ru/news/42897/

Автор: Константин Костенко (Гремячинск).

Средняя оценка: -0.06

Комментарии


97. Вопрос 5.1: Персонаж скандинавских мифов великан СУттунг, желая отомстить своим врагам карликам, отвёз их на маленький островок. Назовите то, что должно было стать причиной их гибели.

Ответ: прилив.

Зачёт: по слову «прилив» и его производным.

Источник: Петрухин В. Мифы древней Скандинавии (http://goo.gl/iEMVJI).

Автор: Сергей Спешков.

Средняя оценка: -0.07

Комментарии


98. Вопрос 2.32: ЭТО стало названием заведения, предлагающего посетителям Бильярд, Бар и Боулинг. Один кавалер ордена Улыбки вспоминал, как ЭТО торчало из рукава его полушубка и рассматривало прохожих. Назовите этого кавалера.

Ответ: Сергей Образцов

Зачёт: По фамилии

Комментарий: Заведение названо по первым двум буквам каждого слова - БИльярд, БАр, БОулинг. Би-Ба-Бо (ударение на О) - это простейшая перчаточная кукла. Орден улыбки вручается людям, радующим детей. Помимо знаменитого кукловода этим орденом награждались Котёночкин, Михалков, Барто и другие.

Источник: 1) http://ru.wikipedia.org/wiki/Би-ба-бо 2) http://www.bibabo.ru/ 3) http://ru.wikipedia.org/wiki/Орден_Улыбки

Автор: Евгений Ярков (Тюмень)

Средняя оценка: -0.07

Комментарии


99. Вопрос 4.29: Согласно одному сайту, у участников ИКСА синхронизируется сердцебиение. Некоторые ИКСы используют технику цепного дыхания. Что мы заменили сло-вом «ИКС»?

Ответ: Хор.

Комментарий: Техника цепного дыхания заключается в том, что в процессе пения хори-сты осуществляют смену дыхания не одновременно. В результате достигается непре-рывность звучания.

Источник:
    1. http://esquire.ru/findings-91
    2. http://kuzenkovd.ru/yutanovka/priemyi-zvukovedeniya-tsepnoe-dyihanie-tembr-golosa-horistov/

Автор: Ярослав Косарев (Нижний Новгород).

Средняя оценка: -0.08

Комментарии (1)


100. Вопрос 5.11: Джозеф Аддисон, публицист 18 века, активно выступал против изобретателей, которых он называл идеалистами. Он с ужасом представлял себе купол собора святого Павла, усеянный людьми. Одно из слов в этом вопросе было немного изменено. Напишите его в исходном виде.

Ответ: дедалистами.

Зачёт: точный ответ.

Комментарий: Аддисон не приветствовал развитие воздухоплавания, и его воображение рисовало ужасные картины мира, в котором люди научатся летать.

Источник: Trouble Above // Futility Closet. — 27.04.2013 (http://goo.gl/Bhf4ys).

Автор: Алексей Бороненко (Челябинск — Москва).

Средняя оценка: -0.08

Комментарии


101. Вопрос 1.19: Терри Пратчетт называет созданный им Плоский мир ЭТИМ для других миров. А героиня Пратчетта использовала несколько экземпляров ЭТОГО для почти беспредельного умножения своей волшебной силы. Назовите ЭТО.

Ответ: зеркало.

Зачёт: точный ответ.

Комментарий: героиня ставила зеркала напротив друг друга, чтобы в них отражалось их бесконечное количество.

Источник: 1)http://royallib.ru/read/pratchett_terri/posledniy_kontinent.html#0 2)www.modernlib.ru/books/pratchett_terri/vedmi_za_granicey/read 3)http://ru.wikipedia.org/wiki/Ведьмы_за_границей

Автор: Константин Костенко (Гремячинск).

Средняя оценка: -0.09

Комментарии


102. Вопрос 3.14: Ю НЕсбё называет ЕЁ серийной убийцей. Одна из первых ЕЁ жертв – Дадли Вольф, искатель приключений и миллионер. Назовите ЕЁ двумя или тремя символами.

Ответ: К2.

Зачёт: К-2.

Комментарий: ОНА – гора К2 (Чогори), которая чуть ниже Эвереста, но значительно опаснее.

Источник:
    1. Несбё Ю. Леопард http://flibusta.net/b/246847/read
    2. http://sevich.livejournal.com/93549.html
    3. http://www.touristclub.narod.ru/k2_history.htm

Автор: Александр Коробейников (Саратов)

Средняя оценка: -0.1

Комментарии


103. Вопрос 3.7: Англо-саксонами один толковый словарь называет шумных американцев. В одном из слов предыдущего предложения мы заменили две буквы одной. Восстановите исходный вид этого слова.

Ответ: англо-клаксонами.

Зачёт: англо-клаксон/ы, клаксон/ы/ами.

Комментарий: постоянно гудят.

Источник: http://www.urbandictionary.com/define.php?term=anglo-klaxon&defid=7231952

Автор: Владислав Декалов (Волгоград)

Средняя оценка: -0.11

Комментарии


104. Вопрос 5.7: В притче Германа МЕлвилла человек соглашается принять у себя возможного преступника. Попутно человек узнаёт, что этот возможный преступник, по крайней мере, не является ИМ. Кем?

Ответ: фальшивомонетчиком.

Зачёт: точный ответ.

Комментарий: человек берёт с возможного преступника мзду и при этом проверяет, не останется ли он сам в накладе.

Источник: Мелвилл Г. Моби Дик, или Белый кит (http://goo.gl/npwKrq).

Автор: Сергей Спешков.

Средняя оценка: -0.12

Комментарии


105. Вопрос 4.20: На одной юмористической картинке Леонардо, полный решимости отомстить за сороди-чей, встречает ЕГО. Назовите ЕГО.

Ответ: Марио.

Зачёт: Водопроводчик Марио; Mario.

Комментарий: Как известно, водопроводчик Марио убивает черепашек за деньги и гри-бочки. На картинке в канализации его находит Леонардо, один из черепашек ниндзя.

Источник: http://www.deltaattack.com/2011/08/26/koopas-get-help-to-fight-mario/

Автор: Ярослав Косарев (Нижний Новгород).

Средняя оценка: -0.12

Комментарии


106. Вопрос 4.32: Главный герой фильма «Здесь курят» буквально проповедует курение. На по-стере этого фильма ПЕРВОЕ выступает в роли ВТОРОГО. Назовите ПЕРВОЕ и ВТОРОЙ.

Ответ: Колечко дыма, нимб.

Зачёт: Кольцо дыма, нимб.

Источник:
    1. Фильм «Здесь курят».
    2. http://st.kinopoisk.ru/images/film_big/87477.jpg

Автор: Ярослав Косарев (Нижний Новгород).

Средняя оценка: -0.12

Комментарии (1)


107. Вопрос 2.27: Однажды, хоть этот день мог никогда не наступить, Гай Пелли оказал своему близкому другу услугу – организовал мальчишник. В этой связи британская пресса недавно называла Пелли одним из главных претендентов на роль ЕГО. Назовите ЕГО двумя словами.

Ответ: Крестный отец

Зачёт: Точный ответ

Комментарий: Начало вопроса является парафразом знаменитой реплики из одноименного романа Марио Пьюзо: "Однажды, хоть этот день может никогда не наступить, я попрошу оказать мне услугу, а до тех пор прими это, как подарок в день свадьбы моей дочери".

Источник: 1) http://news.rambler.ru/20866774/ 2) Фильм “Крестный отец”

Автор: Николай Пручковский (Одесса)

Средняя оценка: -0.13

Комментарии


108. Вопрос 3.24: Мыслитель XVI века МигЕль СервЕт написал трактаты «Об ошибках троичности» и «Восстановление христианства». В романе ЧЕслава МИлоша Сервет перед смертью проклинает городские власти Женевы, поскупившиеся на НИХ. Назовите ИХ одним словом.

Ответ: дрова.

Зачёт: поленья.

Комментарий: сочинения Сервета были признаны еретическими (ещё бы, утверждать, что учение о Троице – ошибочно!), и его сожгли на костре. Сервет известен как первая жертва протестантского фанатизма.

Источник:
    1. http://ru.wikipedia.org/wiki/Сервет,_Мигель
    2. Милош Ч. Долина Иссы http://flibusta.net/b/281622/read

Автор: Александр Коробейников (Саратов)

Средняя оценка: -0.13

Комментарии


109. Вопрос 3.30: Журнал GEO [гео] называет чёрную дыру космическим ИМ. В сказке 1955 года ОН начал с занавесок. Назовите ЕГО.

Ответ: пылесос.

Комментарий: чёрная дыра засасывает в себя материю. Пылесос под управлением Карлсона первым делом попытался засосать в себя тюлевые занавески.

Источник:
    1. GEO. 2013. №4. С. 36.
    2. http://lib.ru/LINDGREN/malysh.txt
    3. http://sv.wikipedia.org/wiki/Astrid_Lindgren

Автор: Александр Коробейников (Саратов)

Средняя оценка: -0.13

Комментарии


110. Вопрос 4.27: Согласно одному сайту, карточная масть «бубны» символизирует шляпки гвоздей, а «червы» - губку. Назовите двумя словами то, с чем на этом сайте ассоциируются «пики».

Ответ: Копье Лонгина.

Источник: http://bania.drezna.ru/content/news.php?id=20

Автор: Ярослав Косарев (Нижний Новгород).

Средняя оценка: -0.13

Комментарии


111. Вопрос 2.22: Скальды называли ИХ стеной корабля. В произведении Плутарха ОНИ упоминаются при описании пышного надгробия девы-предательницы. Назовите ИХ.

Ответ: Щиты

Зачёт: Щит.

Комментарий: Бока драккаров украшали щиты викингов. Предательница Тарпея попросила бросить ей всё, что носят на левых руках солдаты сабинца Татия, и вместе с золотыми браслетами была погребена под щитами воинов.

Источник: 1) http://www.kulichki.com/tolkien/kaminzal/edda/edda2.html 2) http://www.lib.ru/POEEAST/PLUTARH/tesej.txt

Автор: Евгений Ярков (Тюмень)

Средняя оценка: -0.13

Комментарии


112. Вопрос 1.32: Природная ОНА известна с древнейших времён. В XVIII веке ОНА была искусственно получена в одной из пивоварен английского города Лидса. В спорах о ЕЁ вреде для здоровья речь часто идет о сладкой ЕЁ разновидности. Назовите ЕЁ.

Ответ: газированная вода.

Зачёт: газировка.

Комментарий: при пивном брожении выделялся углекислый газ, который попадал в контейнеры с водой. Сладкую газированную воду многие врачи обвиняют в распространении ожирения и диабета.

Источник: 1)http://www.critical.ru/calendar/Priestley.htm 2)http://ru.wikipedia.org/wiki/Газированная_вода 3)http://kaktrostinka.ru/sovetidoktora/gazirovka.html 4)http://www.godembassy.org/main/na-zdorove/item/2912-vred-gazirovannoy-vodyi.html 5)http://domashniy-medic.ru/5-prichin-skazat-gazirovannoj-vode-net

Автор: Константин Костенко (Гремячинск).

Средняя оценка: -0.14

Комментарии


113. Вопрос 4.2: Героя современного романа поместили в психиатрическую клинику, располо-женную на острове. Однако ему удалось сбежать, воспользовавшись ИКСОМ своего товарища. ИК-СЫ описываются в первой главе другого более известного романа. Назовите ИКС.

Ответ: Гроб.

Комментарий: Сюжет содержит аллюзию на побег графа Монте-Кристо. Более известный роман – «12 стульев».

Источник:
    1. http://lib.rus.ec/b/187283/read
    2. http://lib.ru/ILFPETROV/author12.txt

Автор: Сергей Лобачев (Нижний Новгород).

Средняя оценка: -0.15

Комментарии


114. Вопрос 3.5: В материале журнала "Популярная механика", посвящённом новейшим истребителям, говорится о том, что их применение в современных боевых условиях неоправданно. Заголовок материала состоит из трёх слов, второе из которых – «пятое». Воспроизведите два остальных слова, являющиеся устойчивым термином.

Ответ: потерянное поколение.

Комментарий: статья называется "Потерянное пятое поколение" и посвящена современным истребителям, слишком сложным для управления.

Источник: http://www.top-journals.com/2013/07/popularnaya-mexanika-8-avgust-2013.html

Автор: Владислав Декалов (Волгоград)

Средняя оценка: -0.15

Комментарии


115. Вопрос 1.31: Герой Евгения Гришковца утверждал, что в сибирских городах вам покажут дома с необычно большими окнами и скажут, что вот так ОНИ прошли по Сибири. К НИМ можно отнести и некоторых российских правителей. Кто ОНИ?

Ответ: немцы.

Зачёт: точный ответ.

Комментарий: эти дома строили немцы после Второй мировой войны. Несколько российских монархов были этническими немцами, кто частично, а кто и полностью.

Источник: 1)Е. Гришковец. Реки. С. 23. 2)http://ru.wikipedia.org/wiki/Российские_немцы

Автор: Александр Пономарев (Пермь).

Средняя оценка: -0.15

Комментарии


116. Вопрос 5.27: У человека и шимпанзе — общий предок. На это в частности указывают идентичные бесполезные элементы в геномах. Биолог Егор БазЫкин рассказывал об этом школьникам и привёл аналогию, в которой упомянул ИХ. Назовите ИХ двумя словами, которые начинаются на одну и ту же букву.

Ответ: одинаковые опечатки.

Зачёт: одинаковые ошибки, одинаковые описки.

Комментарий: Лектор привёл аналогию, понятную для школьников: если учитель видит одинаковые опечатки в рефератах разных учеников, он понимает, что эти работы списаны из интернета, то есть у них — один и тот же источник.

Источник: Лекция Г.А. Базыкина на «Дне Просветителя» в Московском зоопарке (16.11.2013).

Автор: Алексей Бороненко (Челябинск — Москва).

Средняя оценка: -0.17

Комментарии


117. Вопрос 2.2: В девиз Восточного Тимора входит слово “Прогресс”. Примечательно, что на новом гербе Восточного Тимора вместо стрел и сабель появился ОН. ЕГО предпочёл взять РЭмбо, готовясь к диверсионной операции во Вьетнаме. Назовите ЕГО двумя словами.

Ответ: Автомат Калашникова.

Комментарий: С 2007 года автомат Калашникова изображен на гербе Восточного Тимора, над девизом «Единство, Действие, Прогресс». Рэмбо предпочёл автомат, которым были вооружены вьетнамцы, чтобы не отличаться от них звуком выстрела в перестрелках и при случае пользоваться добытыми у противника патронами.

Источник: 1) http://ru.wikipedia.org/wiki/Герб_Восточного_Тимора
    2) http://detektivi.net/detektivi/chitat/morrell_dyevid_ryembo_2_ryembo-2/morrell_dyevid_ryembo_2_ryembo-2_GG_13.php

Автор: Сергей Коротков (Киев)

Средняя оценка: -0.19

Комментарии


118. Вопрос 2.3: В работе Харро фон Зенгера отмечается, что в западных правовых государствах, с их склонностью тотального регулирования, таится опасность одичания правопорядка. В этой связи Фон Зенгер, упоминая ИХ, отмечает, что кустарники параграфов разрастаются до такой степени, что даже днем царит полумрак. Назовите ИХ двумя равными по количеству букв словами.

Ответ: Джунгли законов

Зачёт: Точный ответ

Комментарий: Тем самым, мнение Харро фон Зенгера фактически выражает афоризм Яцека Вейроха, согласно которому “В джунглях законов расцветает закон джунглей”. Пуант “равным по количеству букв” не только отсекает неправильное “Закон джунглей”, но и дает небольшую подсказку.

Источник: 1) http://ru.wikiquote.org/wiki/Закон_джунглей 2) http://www.libma.ru/istorija/stratagemy_o_kitaiskom_iskusstve_zhit_i_vyzhivat_tt_1_2/p27.php

Автор: Евгений Ярков (Тюмень)

Средняя оценка: -0.21

Комментарии


119. Вопрос 1.16: Прослушайте немного измененное начало статьи: «Руслан Панкратов хочет стать Русланом Панкратовым в соответствии с собственным ощущением национальной идентичности». Из двух слов этого предложения автор Вопроса убрал две одинаковые буквы. Как эти два слова выглядели изначально?

Ответ: Русланс Панкратовс

Зачёт: точный ответ.

Комментарий: дело было в Латвии. И захотел Русланс Панкратовс стать Русланом Панкратовым.

Источник: http://rus.delfi.lv/news/daily/latvia/article.php?id=13782736.

Автор: Александр Пономарев (Пермь).

Средняя оценка: -0.21

Комментарии


120. Вопрос 1.15: В книгах о волшебнике-подростке Гарри Поттере самым авторитетным игроком его команды по квиддичу был Оливер Вуд. Знакомый автора Вопроса, рассуждая об этом игроке, упомянул писателя. Что это за писатель?

Ответ: Жюль Верн.

Зачёт: точный ответ.

Комментарий: Оливер Вуд стал капитаном команды примерно в 15 лет, а у Жюля Верна есть роман «Пятнадцатилетний капитан».

Источник: 1)http://ru.harrypotter.wikia.com/wiki/Оливер_Вуд 2)http://ru.wikipedia.org/wiki/Верн,_Жюль

Автор: Константин Костенко (Гремячинск).

Средняя оценка: -0.22

Комментарии (1)


121. Вопрос 4.8: В одном рассказе, политический деятель, взойдя на трибуну, уперся лбом в НЕГО, нахо-дившееся в полукилометре от места событий. Во многих компьютерных играх ЕГО вид можно настраивать. Назовите ЕГО двумя словами, начинающимися на одну и ту же букву.

Ответ: Перекрестье прицела.

Источник: http://lib.rus.ec/b/371529/read

Автор: Сергей Лобачев (Нижний Новгород).

Средняя оценка: -0.23

Комментарии


122. Вопрос 4.18: В 2004 году ОН на некоторое время перестал выходить на связь. Статья на сайте gazeta.ru [газета точка ру], посвящённая этому событию, была преждевременно озаглав-лена словами – «ОН УМЕР». Какие два слова мы заменили словом «УМЕР»?

Ответ: Испустил дух.

Комментарий: Речь в вопросе идет о марсоходе Spirit [Спирит].

Источник: http://www.gazeta.ru/2004/01/23/marsohodSpir.shtml

Автор: Ярослав Косарев (Нижний Новгород).

Средняя оценка: -0.23

Комментарии


123. Вопрос 3.12: Историки ВалЯнский и КалЮжный пишут, что в эпоху заселения Европы МЕНЕ и ТЕКЕЛ уже и значило ФАРЕС. Что мы заменили словами МЕНЕ, ТЕКЕЛ, ФАРЕС?

Ответ: пришёл, увидел, победил.

Зачёт: veni, vidi, vici; вени, види, вици/вики; прийти, увидеть, победить.

Комментарий: свободных мест было много, и для того, чтобы их занять, сражаться не приходилось. Валянский и Калюжный, конечно, те ещё историки, но эта конкретная мысль довольно разумна.

Источник: Валянский С., Калюжный Д. Явление Руси. М., 1998. С. 67.

Автор: Александр Коробейников (Саратов)

Средняя оценка: -0.24

Комментарии


124. Вопрос 5.29: В рекламе производителя программного обеспечения и компьютерной техники известный персонаж говорит, что его действия были вызваны ЕЮ. И перекладывает вину на людей, которые этого не предусмотрели. Назовите ЕЁ.

Ответ: проблема 2000.

Зачёт: ошибка 2000, millennium bug, миллениум баг, Y2K.

Комментарий: рекламный ролик был размещён на сайте «Apple» в 1999 году. Компьютер HAL [хэл] 9000 из фильма «2001 год: Космическая одиссея», убивший бОльшую часть экипажа, объясняет, что его действия были вызваны «ошибкой 2000». И говорит, что пользователям «Макинтошей» эти проблемы не страшны.

Источник: Arthur C. HAL confesses all and joins Apple // The Independent. — 25.01.1999 (http://goo.gl/7xlkfc).

Автор: Сергей Спешков.

Средняя оценка: -0.24

Комментарии


125. Вопрос 4.3: В статье Википедии о нашем современнике говорится, что его бабушка и мать владели школой, в которой каждый ученик имел возможность изучать то, что ему интересно. Назовите этого современника.

Ответ: Джи́мми Уэ́йлс.

Зачёт: Джи́мбо; Джимми «Джимбо» Уэйлс.

Комментарий: Уэйлс - основатель Википедии. Каждый человек может зайти на Википедию и узнать то, что ему интересно.

Источник: http://ru.wikipedia.org/wiki/Джимми_Уэйлс

Автор: Ярослав Косарев (Нижний Новгород).

Средняя оценка: -0.25

Комментарии


126. Вопрос 3.6: Герои фильма МакОто СинкАя первый раз встречаются в парковой беседке. Поскольку ИКС выдался слишком длинным, то в дальнейшем они встречаются там ещё неоднократно. Назовите ИКС двумя словами.

Ответ: сезон дождей.

Зачёт: точный ответ.

Комментарий: герои встречаются в беседке, где пережидают дождь - в дальнейшем, так как сезон дождей продолжается, они продолжают видеться на том же месте.

Источник: м/ф “Сад слов”, реж. М. Синкай, 2013 г.

Автор: Артём Викторов (Саратов)

Средняя оценка: -0.26

Комментарии


127. Вопрос 1.4: Считается, что коллектив можно назвать ИМ, когда в ЕГО составе не менее двух человек делают почти одно и то же. Общее количество ЕГО участников обычно гораздо больше. А происхождение ЕГО названия вообще ведется от слова, означающего толпу. Назовите ЕГО.

Ответ: хор.

Зачёт: точный ответ.

Комментарий: общепринято, что отличие хора от вокального ансамбля в том, что как минимум два человека исполняют одну и ту же партию. Слово «хор» происходит от греческого «хорос», что означает «толпа».

Источник: http://ru.wikipedia.org/wiki/Хор

Автор: Александр Пономарев (Пермь).

Средняя оценка: -0.26

Комментарии


128. Вопрос 1.35: Один киногерой добился расположения у нескольких жителей крупной азиатской страны, разрешив им называть себя более коротко. Что это за киногерой?

Ответ: Индиана Джонс.

Зачёт: Генри Джонс-младший.

Комментарий: он разрешил индийцам называть его Инди.

Источник: Хроники молодого Индианы Джонса. 5 серия.

Автор: Константин Костенко (Гремячинск).

Средняя оценка: -0.27

Комментарии


129. Вопрос 5.22: 4 октября 1937 года после очередной попытки реанимировать своё творение разуверившийся драматург произнёс два слова. Напишите их.

Ответ: «Бег» умер.

Зачёт: бег умер.

Комментарий: судьба пьесы была очень незавидна. Булгаков несколько раз возвращался к попыткам поставить её. После очередной неудачи он отреагировал созвучно фразе Ницше «Бог умер».

Источник: Воспоминания о Михаиле Булгакове: Е. С. Булгакова, Т. Н. Лаппа, Л. Е. Белозерская. — М.: АСТ : Астрель, 2006.

Автор: Сергей Спешков.

Средняя оценка: -0.27

Комментарии


130. Вопрос 5.8: Литературный агент ГАрольд СвОнсон остроумно заметил, что наибольшую прибыль может принести составление ИХ. В одной статье с НИМИ сравнивается плохо оформленный сайт. Назовите ИХ.

Ответ: записки с требованием выкупа.

Зачёт: по словам «требование выкупа».

Комментарий: типовой приём в кино и литературе — похититель вырезает из разных газет буквы и вклеивает их на один лист. «Эффект записок с требованием выкупа» — термин в типографском деле, описывающий использование множества разных шрифтов в одном документе.

Источник:
    1. QI: some quite interesting facts about writing (http://goo.gl/yAc1vC).
    2. http://en.wikipedia.org/wiki/Ransom_note
    3. What is Ransom Note Desktop Publishing? (http://goo.gl/eO3irh).

Автор: Алексей Бороненко (Челябинск — Москва).

Средняя оценка: -0.29

Комментарии


131. Вопрос 2.15: [Внимание, в некоторых словах вопроса необходимо будет делать паузы. При этом, слово “пауза” в квадратных скобках произносить не нужно]
    Персонаж Сигизмунда Кржижа [пауза] новского, коллекционировавший ИХ, сравнивал ИХ с бабочками, порха [пауза] ющими с цветка на цветок. Присутствуя на различных лекциях, этот персо [пауза] наж всегда садился к докладчикам спиной. Назовите ИХ.

Ответ: Зевки

Зачёт: Точный ответ

Комментарий: Зевота, как известно, заразна, а на скучных лекциях люди часто зевают. Паузы в некоторых словах даны в качестве небольшой подсказки.

Источник: http://az.lib.ru/k/krzhizhanowskij_s_d/text_0270.shtml

Автор: Евгений Ярков (Тюмень)

Средняя оценка: -0.31

Комментарии


132. Вопрос 5.23: Герой фильма «Парень из нашего города» попадает в плен. Он безуспешно пытается выдать себя за европейца, и допрашивающий по этому поводу произносит саркастическую реплику. Назовите язык и город, которые упоминаются в этой реплике.

Ответ: французский язык, Нижний Новгород.

Зачёт: французский, Горький.

Комментарий: «А французскому языку его обучили где-нибудь в Нижнем Новгороде?!»

Источник: Парень из нашего города, реж. А. Столпер, Б. Иванов, 1942 г.

Автор: Сергей Спешков.

Средняя оценка: -0.38

Комментарии


133. Вопрос 4.33: Джонни Мосс начал играть в 10 лет и стал настоящим асом. Поэтому в честь него назы-вают ЕЁ. Назовите ЕЁ как можно точнее.

Ответ: Комбинация десятки и туза.

Зачёт: По упоминанию десятки и туза без неверных уточнений.

Комментарий: Джонни Мосс - профессиональный игрок в покер.

Источник: http://en.wikipedia.org/wiki/List_of_playing-card_nicknames

Автор: Ярослав Косарев (Нижний Новгород).

Средняя оценка: -0.4

Комментарии


134. Вопрос 3.1: В первой части киноцикла «ДекалОг» герои слишком доверяют своим компьютерам. Какое требование из четырёх слов предложил для этой части пользователь сайта kinopoisk.ru [кинопоиск точка ру]?

Ответ: «Не сотвори себе кумира».

Зачёт: точный ответ в кавычках или без них.

Комментарий: «Декалог» состоит из десяти фильмов, примерно соответствующих десяти библейским заповедям.

Источник: http://www.kinopoisk.ru/user/5780/comment/170469/

Автор: Александр Коробейников (Саратов)

Средняя оценка: -0.4

Комментарии


135. Вопрос 2.28: На иллюстрации к статье "Идеальный Янукович" изображён ОН с непропорционально большой головой президента Украины. Назовите ЕГО двумя словами.

Ответ: Витрувианский человек.

Комментарий: В соответствии с сопроводительными записями Леонардо, рисунок "Витрувианский человек" был создан для определения пропорций мужского тела, как оно описано в трактатах античного римского архитектора Витрувия. Слова "непропорционально большой" - подсказка.

Источник: ЛОАВ

Автор: Сергей Коротков (Киев)

Средняя оценка: -0.44

Комментарии


136. Вопрос 2.31: В одной пьесе ПЕРВЫЙ знакомится со ВТОРЫМ и радостно просит у него ухИ, однако ВТОРОЙ замечает, что ПЕРВЫЙ обознался. В конце жизни ВТОРОЙ вынужден был продать свою библиотеку. Кого мы заменили словами «ПЕРВЫЙ» и «ВТОРОЙ»?

Ответ: Михаил Голодный, Демьян Бедный

Зачёт: По фамилиям без неверных уточнений, в любом порядке.

Комментарий: Поэт Михаил Голодный знакомится с Демьяном Бедным и, чувствуя голод, просит Демьяновой ухи, но Бедный замечает, что он – не тот Демьян. В конце жизни Демьян Бедный был действительно беден. Библиотека у Бедного была знатная.

Источник: 1) http://ru.wikipedia.org/wiki/Демьян_Бедный 2) http://lit.lib.ru/k/krasnogorow_w_s/luchshe_netu_togo_svetudoc.shtml

Автор: Евгений Ярков (Тюмень)

Средняя оценка: -0.45

Комментарии


137. Вопрос 5.2: Антон ДОлин рассказывает, как на одном показе «Звёздных войн» фанаты использовали бутафорские ПЕРВЫЕ в том же качестве, в котором уже почти полвека используются ВТОРЫЕ. Назовите ПЕРВЫЕ и ВТОРЫЕ.

Ответ: лазерные мечи, зажигалки.

Зачёт: световые мечи или мечи - вместо лазерных мечей. В любом порядке.

Комментарий: подобно фанатам на рок-концерте, поклонники «Звёздных войн» размахивали лазерными мечами в воздухе. Традиция с зажигалками началась на легендарном фестивале в Вудстоке.

Источник:
    1. Долин А. Слабая сторона Силы (http://goo.gl/WeyqBm).
    2. Alice M. What band started the tradition of holding up lighters // San Diego Reader. — 29.08.2002 (http://goo.gl/IpTEYq).

Автор: Алексей Бороненко (Челябинск — Москва).

Средняя оценка: -0.45

Комментарии


138. Вопрос 5.30: На картине ЛУкаса КрАнаха Старшего Купидон жалуется на укусы пчёл. СтефАно ДзУффи пишет, что КрАнах таким образом предостерегает людей от нарушения заповеди. С чем ДзУффи сравнивает последствия от укусов?

Ответ: с венерическими заболеваниями.

Зачёт: с венерическими болезнями, с заболеваниями, передающимися половым путём.

Комментарий: Купидон решил полакомиться мёдом и получил ожидаемую расплату, на что жалуется матери Венере. Исследователь полагает, что это моральное предостережение против рисков, связанных со сладострастием. Что было особенно актуально для средневековой Европы с её невысоким уровнем медицины.

Источник: Дзуффи С. Ренессанс: история, эстетика, мастера, шедевры. — М.: «Омега», 2008.

Автор: Сергей Спешков.

Средняя оценка: -0.45

Комментарии


139. Вопрос 2.18: В стихотворении Гийома дю Вентре, описывающем снежное утро, мороз начищает лунный диск до блеска, а львы у Темзы надевают ИХ. В одной компьютерной игре Индиана Джонс дарит ЕЁ персонажу для того, чтобы было удобнее следить за ним в толпе. Назовите ЕЁ.

Ответ: Феска

Зачёт: Точный ответ

Комментарий: Снег образовал на головах львов “шапочки”, напоминающие турецкие фески. Рифма “блеска-феска”. Традиционная красная феска выделяет персонажа из толпы.

Источник: 1) http://www.sakharov-center.ru/asfcd/auth/?t=page&num=4003 2) http://ru.wikipedia.org/wiki/Indiana_Jones_and_the_Fate_of_Atlantis

Автор: Евгений Ярков (Тюмень)

Средняя оценка: -0.48

Комментарии


140. Вопрос 1.10: Одним из параметров ЕЁ электрической разновидности является крутящий момент. Производительность может превышать килограмм в минуту, а избежать наматывания можно с помощью автореверса. Назовите ЕЁ.

Ответ: мясорубка.

Зачёт: точный ответ.

Источник: 1)http://ru.wikipedia.org/wiki/Мясорубка 2)http://kulinarnye-sovety.ru/spravochnik-kulinara/1389-vybiraem-elektricheskuyu-myasorubku.html 3)http://bt.rozetka.com.ua/ru/products/procategory/298/index.html

Автор: Александр Пономарев (Пермь).

Средняя оценка: -0.48

Комментарии


141. Вопрос 2.29: Андский канделябр, расположенный неподалёку он перуанского города Писко, набожные испанцы называли ИМИ. Отважный герой одной песни, подгоняя лошадь, рассчитывает на НИХ Георгия. Назовите ИХ двумя словами.

Ответ: Три креста

Зачёт: Точный ответ

Комментарий: Андский канделябр внешне напоминает украинский трезубец. Вполне логично, что его сравнивали с тремя крестами, расположенными на Голгофе. Аллюр “три креста” - самый быстрый бег лошади для выполнения срочного задания.

Источник: 1) http://lit.lib.ru/m/mashkowcew_w_i/text_0040.shtml 2) http://ru.wikipedia.org/wiki/Георгиевский_крест 3) http://ru.wikipedia.org/wiki/Андский_канделябр

Автор: Евгений Ярков (Тюмень)

Средняя оценка: -0.49

Комментарии


142. Вопрос 4.9: В одной биографической книге о НЁМ описан случай, когда к ЕГО матери при-шел друг семьи и сказал: «Я оплатил вексель... и теперь это мой дом». Назовите ЕГО.

Ответ: [Антон Павлович] Чехов.

Зачёт: По фамилии без неверных уточнений.

Комментарий: Эта история напоминает эпизод, произошедший в финале пьесы «Вишнё-вый сад».

Источник: Дональд Рейфилд «Жизнь Антона Чехова».

Автор: Ярослав Косарев (Нижний Новгород).

Средняя оценка: -0.52

Комментарии


143. Вопрос 3.23: Дэвид КрОненберг, чтобы подчеркнуть растущую абстрактность современного мира, использовал в начальных титрах одного своего фильма стилизацию ПРОПУСК. Восстановите три пропущенных слова, каждое из которых начинается на две одинаковых буквы.

Ответ: под полотна Поллока.

Зачёт: под поллоковские полотна.

Комментарий: чьи работы еще использовать для иллюстрации абстрактности, как не картины виднейшего представителя абстрактного экспрессионизма.

Источник: http://ru.wikipedia.org/wiki/Космополис_(фильм)

Автор: Владислав Декалов (Волгоград)

Средняя оценка: -0.53

Комментарии


144. Вопрос 4.1: В одной статье Ашот Джазоян рассказывает о победах в чемпионате и кубке СССР в 1973 году. Заголовок статьи лишь кавычками отличается от названия того, что находится на высоте более чем 5000 метров. Воспроизведите назва-ние статьи.

Ответ: Вершина «Арарата».

Комментарий: Речь в статье идет о футбольном Арарате из Еревана, высшее достижение в истории которого - золотой дубль в 1973 году.

Источник:
    1. Журнал «Огонёк» №34 2013 года стр. 44.
    2. http://ru.wikipedia.org/wiki/Арарат

Автор: Ярослав Косарев (Нижний Новгород).

Средняя оценка: -0.53

Комментарии (2)


145. Вопрос 5.35: Однажды на экзамене по механике немолодой преподаватель спросил Александра Городницкого, почему тот не может СДЕЛАТЬ ЭТО. Взглянув на преподавателя, Городницкий неучтиво ответил: «Вас тоже не могу». Что в этом вопросе заменено словами СДЕЛАТЬ ЭТО?

Ответ: поднять себя за волосы.

Зачёт: по словам «поднять» плюс «себя / Городницкого» плюс «волосы».

Комментарий: профессор спросил, почему Городницкий не может поднять себя за волосы, а преподавателя может. Глядя на лысого профессора, Городницкий резонно заметил, что и его не сможет поднять. По словам Городницкого, зачёт он всё равно получил.

Источник: Городницкий А. Атланты. Моя кругосветная жизнь (http://flibusta.net/b/323596/read).

Автор: Сергей Спешков.

Средняя оценка: -0.53

Комментарии


146. Вопрос 4.17: Многие современные японцы верят, что изменить судьбу можно с помощью пластической операции. Не удивительно, что к этому способу негативно отно-сятся люди, зарабатывающие ЭТИМ. Назовите ЭТО словом греческого проис-хождения.

Ответ: Хиромантия.

Комментарий: Речь идёт об изменении линий на ладонях.

Источник: http://expert.ru/2013/07/17/tvortsyi-sobstvennyih-sudeb/

Автор: Александр Сабреков (Москва).

Средняя оценка: -0.53

Комментарии


147. Вопрос 4.26: Заголовок статьи в журнале GEO, повествующей о работе шаманов, одной буквой отли-чается от карты. Воспроизведите этот заголовок.

Ответ: Бубновый балет.

Комментарий: В работе шаманы используют бубен, при этом сопровождая игру на нем ритуальными танцами.

Источник: Журнал GEO сентябрь 2013 стр. 94.

Автор: Ярослав Косарев (Нижний Новгород).

Средняя оценка: -0.53

Комментарии


148. Вопрос 1.22: Если верить Люси Сигл, в начале XXI века средняя женщина ежегодно покупает 28 килограммов ЕЁ, что в несколько раз больше, чем за два-три десятилетия до этого. По мнению Бориса Крутиера, ОНА чутко следит за ростом человека. Назовите ЕЁ.

Ответ: одежда.

Зачёт: точный ответ.

Комментарий: человек растет, и одежда становится мала.

Источник: 1)http://ivona.bigmir.net/beauty/news/313839-Ezhegodno-zhencshina-pokupaet-28-kilogrammov-odezhdy 2)http://wisdom.kulichki.com/?pt=krutier&pn=8

Автор: Александр Пономарев (Пермь).

Средняя оценка: -0.55

Комментарии


149. Вопрос 2.11: В одном из эпизодов фильма "Опасная гастроль", герой Дольфа Лундгрена, бывший морпех, а ныне рок-музыкант, убивает террориста с помощью одной из НИХ. Назовите ИХ.

Ответ: Барабанные палочки

Зачёт: Точный ответ

Комментарий: Как известно из голливудских фильмов, американские морпехи могут мочить террористов и голыми руками, а уж если есть что-то в руках – тем более. Лундгрен в фильме "Опасная гастроль" сыграл ударника группы "CMF" [си-эм-эф]. Бочонок с числом 11 в лото, как правило, называют барабанными палочками.

Источник: х/ф “Опасная гастроль”

Автор: Николай Пручковский (Одесса)

Средняя оценка: -0.55

Комментарии


150. Вопрос 3.9: Мо Янь сравнивает лицо своей героини с цветком хмеля, а мимолётные воспоминания об этой героине – с НИМИ. Назовите ИХ тремя словами, начинающимися на одну и ту же букву.

Ответ: пузырьки пивной пены.

Зачёт: пузырьки пены пива, пузыри пивной пены/пены пива.

Комментарий: воспоминание было мимолетным и лопнуло, словно пузырьки пивной пены в стакане.

Источник: Мо Янь. Страна вина http://www.loveread.ec/read_book.php?id=18350&p=2

Автор: Артём Викторов (Саратов)

Средняя оценка: -0.55

Комментарии


151. Вопрос 5.24: Писатель Святослав ЛОгинов рассказывает в автобиографическом произведении о времени, когда он работал грузчиком. Напишите название этого произведения.

Ответ: Мои универсамы.

Зачёт: Мои универмаги.

Комментарий: явная параллель с «Моими университетами» Максима Горького.

Источник: Логинов С. Мои универсамы (http://goo.gl/NHqIq).

Автор: Алексей Бороненко (Челябинск — Москва).

Средняя оценка: -0.6

Комментарии (1)


152. Вопрос 5.25: Согласно одному американскому исследованию, человек лучше запоминает название бренда, если непроизвольно его проговаривает. Поэтому автор исследования предположил, что скоро в определённых заведениях перестанут продавать... что?

Ответ: попкорн.

Зачёт: точный ответ.

Комментарий: когда мы жуём, описанный механизм работает значительно хуже. Соответственно, люди, которые в кинотеатре жуют попкорн, менее восприимчивы к рекламе, которая демонстрируется перед просмотром фильма.

Источник: Oltermann P. Eating popcorn in the cinema makes people immune to advertising (http://goo.gl/g4Edcl).

Автор: Алексей Бороненко (Челябинск — Москва).

Средняя оценка: -0.61

Комментарии


153. Вопрос 4.31: [Ведущему: Прочитать слово «Жителям» предельно чётко].
    Медсестра из одного романа влюбилась в раненого солдата. Главврач госпи-таля намерен поскорее выписать этого солдата и в одном разговоре проводит параллель с эпизодом из жизни ИКСА. Жителям какого города ИКС был пред-ставлен в начале 16 века?

Ответ: Флоренция.

Комментарий: ИКС – Давид. Давид влюбился в Вирсавию и отправил её мужа – воина Урию – воевать в первых рядах. Урию убили и Давид смог жениться на Вирсавии.

Источник:
    1. Ю Несбё «Красношейка».
    2. http://ru.wikipedia.org/wiki/Давид_(Микеланджело)

Автор: Ярослав Косарев (Нижний Новгород).

Средняя оценка: -0.61

Комментарии


154. Вопрос 5.20: История про НЕГО так сильно впечатлила персонажа ВУди Аллена, что даже преподнесённую ему пару обуви тот изучал с фонариком. Робот из фильма ШанмугАма ШанкАра сравнивает свою влюблённость с тем, что получило своё название в честь НЕГО. Назовите ЕГО.

Ответ: троянский конь.

Зачёт: точный ответ.

Комментарий: прочтя историю о Троянском коне, персонаж Аллена ко всем подаркам стал относиться к подозрением. Определённый тип вредоносного ПО называют «троянскими программами», или «троянами». Роботам романтические чувства не свойственны, поэтому логично предположить, что это некий сбой в программе.

Источник: .
    1. Allen W. The Irish Genius // Without Feathers. Ballantine Books, 1983. P. 125.
    2. Робот (Endhiran), реж. Ш. Шанкар, 2010 г.
    3. http://ru.wikipedia.org/wiki/Троянская_программа

Автор: Алексей Бороненко (Челябинск — Москва).

Средняя оценка: -0.63

Комментарии


155. Вопрос 5.19: В романе КамЮ из-за эпидемии чумы в городе прекращаются все сообщения с внешним миром. Люди мечтают о том времени, когда они смогут вернуться к нормальной жизни и встретиться со своими близкими. Камю называет город одним большим ИМ. Назовите ЕГО.

Ответ: зал ожидания.

Зачёт: точный ответ.

Комментарий: жители города ждали, когда можно будет вернуться к нормальной жизни.

Источник: Камю А. Чума (http://goo.gl/QAkJRW).

Автор: Сергей Спешков.

Средняя оценка: -0.67

Комментарии


156. Вопрос 3.31: Японцы называют ИХ "дорАма". По распространённой версии, ОНИ обязаны своим названием известной компании. Назовите ИХ двумя словами.

Ответ: мыльные оперы.

Зачёт: soap operas.

Комментарий: слово “дорама” созвучно русскому слову “драма” (японцы не произносят два согласных звука подряд). По одной из версий, мыльные оперы спонсировались компанией «Procter & Gamble».

Источник:
    1. http://ru.wikipedia.org/wiki/Дорама
    2. http://ru.wikipedia.org/wiki/Procter_%26_Gamble

Автор: Артём Викторов, Александр Коробейников (оба – Саратов)

Средняя оценка: -0.7

Комментарии


157. Вопрос 3.3: Ричард Фейнман в юности вместе с Томом РутисхАузером и Ральфом ЛЕйтоном создал музыкальную группу. Её название Фейнман позаимствовал из того же литературного произведения, что и другой известный физик. Напишите это название из двух слов.

Ответ: «Три кварка».

Зачёт: Three Quarks; наличие кавычек не принципиально.

Комментарий: Фейнман, подобно Мюррею Гелл-Ману, который ввёл в физику понятие кварка, взял это название из стихотворного отрывка романа Джойса «Поминки по Финнегану».

Источник:
    1. http://en.wikipedia.org/wiki/Quark
    2. Фейнман Р. Вы, конечно, шутите, мистер Фейнман! http://lib.ru/ANEKDOTY/FEINMAN/feinman.txt_with-big-pictures.html
    3. http://ru.wikipedia.org/wiki/Гелл-Ман,_Марри

Автор: Владислав Декалов (Волгоград)

Средняя оценка: -0.7

Комментарии


158. Вопрос 2.34: [На столы раздается текст из файла 34.doc. Раздаточный материал разрезается и ОБЯЗАТЕЛЬНО сворачивается в "трубочку". Текст раздатки не озвучивается]

Раздаточный материал

    - Когда я стану совсем большая, я буду делать все, что хочу. Я буду кормить ядом моих рабов и буду смотреть, как они корчатся.
    - Ну, а сейчас почему ты не дома, не в постели?
    - Потому что сюда идут римляне, и они нас всех съедят. Ты ведь тоже не дома и не в постели.
    - Нет, я дома. Мой дом - палатка.

    Назовите обоих участников розданного вам диалога.

Ответ: [Гай Юлий] Цезарь, Клеопатра,

Зачёт: в любом порядке.

Комментарий: Свернутая в трубочку раздатка – аналог ковра, в котором Клеопатра предстала Цезарю.

Источник: www.world-art.ru/lyric/lyric.php?id=6077

Автор: Евгений Ярков (Тюмень)

Средняя оценка: -0.7

Комментарии


159. Вопрос 3.27: [Ведущему: просьба читать этот вопрос особенно медленно, так как он очень длинный!]
    Порой современный биологический вид путают с вымершим, наделённым похожими признаками. Подобные случаи учёные в шутку называют ПРИНЦИПОМ ПОЛА. В недавней статье ПРИНЦИПОМ ПОЛА назвали ситуацию с японскими диджеями, которые переделывают на свой лад рэгги, причём их записи продаются гораздо лучше оригинальных ямайских. Какие два слова, начинающиеся на одну и ту же букву, мы заменили словами ПРИНЦИП ПОЛА?

Ответ: эффект Элвиса.

Зачёт: точный ответ.

Комментарий: как и в случае со смертью короля рок-н-ролла, которого якобы видели живым в разных точках земного шара, исследователи иногда наблюдают давно вымерших животных, которые оказываются в лучшем случае двойниками. Элвис своих песен не писал, но рок-н-роллы в его исполнении становились намного популярнее, чем песни оригинальных авторов.

Источник:
    1. http://okbox.ru/?p=4368
    2. http://www.rap.ru/reading/1607

Автор: Владислав Декалов (Волгоград)

Средняя оценка: -0.79

Комментарии


160. Вопрос 2.1: Автобиография НОрмана ДУгласа была написана на основе случайно выбранных писателем ИХ. Для изготовления ИХ в девятнадцатом веке использовалась камера, производящая восемь небольших фотографий. Назовите ЕЁ двумя словами.

Ответ: Визитная карточка

Зачёт: Точный ответ

Источник: 1) http://www.proza.ru/2013/08/05/97 2) http://ru.wikipedia.org/wiki/Визитная_карточка

Автор: Евгений Ярков (Тюмень)

Средняя оценка: -0.81

Комментарии


161. Вопрос 1.18: Ричард Фейнман писал, что бразильские муравьи-листоеды не очень умны. Муравью нужно немало потрудиться – подняться на дерево и отгрызть часть листа, чтобы потом унести в муравейник. Во сколько процентов Фейнман оценил вероятность того, что муравей прогрызал лист зря?

Ответ: 50.

Зачёт: точный ответ.

Комментарий: когда муравей догрызает лист до края, ему надо схватиться за отгрызаемый кусок, а не за тот, который прикреплен к ветке. Так как он не очень умен, вероятность этого будет равна одной второй.

Источник: http://aldebaran.ru/zproz/feynman/feynman1/?6

Автор: Александр Пономарев (Пермь).

Средняя оценка: -0.83

Комментарии


162. Вопрос 2.35: [На столы раздается изображение из файла 35.doc. Это изображение необходимо разрезать по контуру и порвать вручную пополам по вертикали примерно в районе букв “i” и “с” в слове “Medications”, “o” и “r” в слове “history”, “i” и “n” в слове “leading”. Указание ведущему: начать чтение со слов “преамбулы” (первые слова - “приглашая авторов”), не обозначая отдельно номер тура и номер вопроса]
    Приглашая авторов и редакторов к работе над открытым всероссийским синхронным чемпионатом, Борис Савельев просил откликнуться тех, кто желает доказать, что может сделать пакет, не получив в ответ отзывы "овощ как овощ", "нормально" или даже "взяли, забыли". Внимание, вопрос номер тридцать пять, раздаточный материал.
    В англоязычных странах офицеры полиции при вызове скорой помощи пользуются специальным правилом для запоминания того, какую информацию о пострадавшем им нужно передать медикам. В состав этой информации входит: описание симптомов, сведения об аллергии, медикаментах, прошедших болезнях, последнем принятии лекарств и событиях, приведших к травме или заболеванию. Каким словом французского происхождения можно перевести название этого правила на русский язык?

Ответ: Шаблон

Зачёт: Точный ответ

Комментарий: Это мнемоническое правило, для которого используются первые буквы соответствующей информации: Symptoms, Allergies, Medications, Past medical history, Last oral intake, Events leading up to the injury - иначе SAMPLE [сЭмпл]. Английское слово sample можно перевести, как шаблон (из франц. échantillon [эшантийОн] - "образчик, проба"). Редакторы так сказать “порвали шаблон”.

Источник: http://en.wikipedia.org/wiki/SAMPLE_history

Автор: Евгений Ярков (Тюмень)

Средняя оценка: -0.9

Комментарии


163. Вопрос 3.19: В настольных играх развитие сюжета зависит не только от ведущего, но и от игроков, поэтому часто задуманная ведущим деталь провисает. По мнению автора вопроса, в этом случае ПЕРВОЕ производит лишь ВТОРОЙ. Назовите ПЕРВОЕ и ВТОРОЙ, двумя словами каждое.

Ответ: ружьё Чехова, холостой выстрел.

Зачёт: висящее/чеховское ружьё, холостой выстрел.

Комментарий: малозначительная деталь, которая потом должна стать важной, таковой из-за действий игроков не становится, поэтому ружьё Чехова производит лишь холостой выстрел.

Источник: http://ru.wikipedia.org/wiki/Ружьё_Чехова

Автор: Артём Викторов (Саратов)

Средняя оценка: -0.91

Комментарии


164. Вопрос 5.31: Достоевский писал, что ужасный климат Женевы очень вреден для его подорванного здоровья. И сравнивал её с КАннами. Какой французский город в этом вопросе заменён на КАнны?

Ответ: КайЕнна.

Зачёт: точный ответ.

Комментарий: бывший каторжанин Достоевский выбрал для нелестного сравнения место, куда во Франции ссылали на каторгу.

Источник: Шишкин М. Русская Швейцария (http://flibusta.net/b/337028/read).

Автор: Сергей Спешков.

Средняя оценка: -0.92

Комментарии


165. Вопрос 3.29: Дмитрий Чернышёв полагает, что благодаря его книге читатели извлекут из себя новые идеи, и сравнивает эту книгу с ИКСОМ. Какой украинский город представляет команда “ИКС”?

Ответ: Кременчуг.

Зачёт: точный ответ.

Комментарий: ИКС - кремень, как при помощи кремня добывают огонь, так и книга поможет читателям разжечь искры новых идей; футбольный клуб “Кремень” логично представляет город Кременчуг. Разумеется, знать о существовании такой команды необязательно.

Источник:
    1. http://www.adme.ru/psihologiya/dmitrij-chernyshev-napisal-knigu-o-tom-kak-dumat-486705/
    2. http://ru.wikipedia.org/wiki/Кремень_(футбольный_клуб)

Автор: Артём Викторов (Саратов)

Средняя оценка: -1.08

Комментарии


166. Вопрос 2.16: Герман Меллвил сравнивал ИХ и с тараном, и с цитаделью, в которой можно выдержать любую осаду времени. Назовите ИХ одним словом.

Ответ: Черепахи

Зачёт: Точный ответ

Источник: http://lib.ru/INPROZ/MELWILL/melvill_islands.txt

Автор: Евгений Ярков (Тюмень)

Средняя оценка: -1.11

Комментарии


167. Вопрос 3.35: В одном рассказе описывается предприимчивый Хуан КапистрАно, которому ОНИ, раз за разом огибая мыс Горн, приносили баснословные барыши. Назовите ИХ двумя словами, начинающимися на одни и те же пять букв.

Ответ: караваны каравелл.

Комментарий: сравнение с торговыми караванами в пустыне; как можно догадаться по имени персонажа, он испанец, а потому использование каравелл логично.

Источник: http://www.lebed.com/2001/art2723.htm

Автор: Артём Викторов (Саратов)

Средняя оценка: -1.11

Комментарии


168. Вопрос 2.25: Большинство из вас и без помощи интернета вспомнят годы жизни Пушкина. А вот один из НИХ представляет собой годы жизни Карамзина. Назовите ЕГО как можно точнее.

Ответ: пароль на WiFi

Зачёт: пароль к/от WiFi. Вайфай можно кириллицей и с незначительными ошибками

Источник: http://pit.dirty.ru/lepro/2/2013/10/16/11556-113533-8ea9f445f05870f0af21e3717f8b033e.jpg

Автор: Николай Пручковский (Одесса)

Средняя оценка: -1.15

Комментарии


169. Вопрос 1.8: ОНИ могут иметь квадратное сечение. Однажды машинист метрополитена Андрей Ульянов врезался в одну из НИХ, а другая пробила дырку в одном из вагонов состава. Назовите ИХ.

Ответ: сваи.

Зачёт: точный ответ.

Комментарий: наверху вбивали сваи и пробили отверстия до туннеля метро. К счастью, никто не пострадал, а Ульянову дали медаль.

Источник: 1)http://ru.wikipedia.org/wiki/Свая 2)http://www.gazeta.ru/2006/05/30/last201642.shtml

Автор: Александр Пономарев (Пермь).

Средняя оценка: -1.16

Комментарии


170. Вопрос 2.36: [Ведущему - чётко прочитать имена Реша, Жуха]
    Домашних любимцев знакомой автора вопроса сокращённо называют РЕша и ЖУха. Назовите их полные имена.

Ответ: Респект, Уважуха.

Зачёт: В любом порядке, с незначительными грамматическими ошибками.

Комментарий: Уважаемые команды! Благодарим за внимание и за участие в нашем турнире.

Источник: ЛОАВ

Автор: Сергей Коротков (Киев)

Средняя оценка: -1.22

Комментарии


171. Вопрос 3.36: Известная модель Грейс Джонс в своём дебютном музыкальном альбоме решила показать себя с лучшей стороны и как певица. Каким словом итальянского происхождения называется этот альбом?

Ответ: Portfolio.

Зачёт: портфолио.

Комментарий: термин "портфолио" чаще используется как раз в модельном бизнесе. Грейс Джонс, обладавшей вокальными данными, было что показать и здесь.

Источник: http://en.wikipedia.org/wiki/Portfolio_(Grace_Jones_album)

Автор: Владислав Декалов (Волгоград)

Средняя оценка: -1.32

Комментарии


172. Вопрос 3.18: Согласно шутке с сайта horofun.com [хорофАн точка ком], ОНИ - это милиционеры, которые всё время кого-то бьют на Красной площади. Назовите ИХ одним словом.

Ответ: куранты.

Зачёт: точный ответ.

Комментарий: тоже бьют на Красной площади.

Источник: http://horofun.com/interesnoe/33-zabluzhdeniya-o-znakomyx-veshhax/

Автор: Александр Коробейников (Саратов)

Средняя оценка: -1.34

Комментарии (1)


173. Вопрос 3.22: В одном сериале описывается боевое построение, позволяющее эффективно избегать столкновений с врагами. Это построение называют живым шабашем. Какое слово мы заменили в этом вопросе?

Ответ: радар.

Зачёт: радаром.

Комментарий: идея строя состоит в том, чтобы вовремя засекать наступающих титанов, тем самым избегая столкновения с ними. Радар и шабаш – палиндромы.

Источник: м/с «Атака титанов», 16-17 серия.

Автор: Артём Викторов (Саратов)

Средняя оценка: -1.36

Комментарии


174. Вопрос 3.15: В статье о племенах бедуинов сообщается, что власть в таких племенах чисто формальна и проявляется только в условиях ограниченных ресурсов. Назовите одним словом то, с чем в статье сравнивается такая власть.

Ответ: мираж.

Зачёт: миражи.

Комментарий: племена бедуинов живут в пустыне, где власть так же призрачна, как и мираж, – многие миражи видятся путникам, страдающим от жажды.

Источник: Discovery. 2013. №7. С. 18.

Автор: Артём Викторов (Саратов)

Средняя оценка: -1.38

Комментарии


175. Вопрос 5.15: В 1960-е годы левые, критикуя внутреннюю политику США, иногда писали название страны по-немецки. В журнале «Black World» [Блэк Уорлд] к этому варианту написания добавили две буквы. Напишите то, что в итоге получилось.

Ответ: AmeriKKKa.

Зачёт: АмериКККа.

Комментарий: намёк на высокий уровень распространения расизма в США.

Источник: http://en.wikipedia.org/wiki/Satiric_misspelling

Автор: Алексей Бороненко (Челябинск — Москва).

Средняя оценка: -1.5

Комментарии


176. Вопрос 2.33:
    В одном произведении фигурирует русский, прошедший сквозь множество тяжелых сражений и выживший даже после приговора к высшей мере наказания. После этого он стал работать в цирюльне под прозвищем, которое можно найти в розданном тексте. Напишите это прозвище.

Ответ: Бритва

Зачёт: Точный ответ

Комментарий: Как и Борис Бритва из фильма “Большой куш” - “Говорят, его вообще невозможно убить”. Впрочем, “попасть в бритву” довольно сложно.

Источник: 1) http://www.lib.ru/NABOKOW/britwa.txt 2) http://citaty.info/film/bolshoi-kush-snatch

Автор: Евгений Ярков (Тюмень)

Средняя оценка: -1.53

Комментарии


177. Вопрос 5.14: Согласно одной шутке, похмелье — это КИЛИМАНДЖАРО СНЕГОВ. Какие два слова мы заменили на КИЛИМАНДЖАРО СНЕГОВ?

Ответ: гнев гроздей.

Зачёт: точный ответ.

Комментарий: каламбурный обыгрыш названия произведения Джона Стейнбека «Гроздья гнева». «Снега Килиманджаро» — произведение другого американского писателя.

Источник: http://en.wikipedia.org/wiki/Transpositional_puns

Автор: Алексей Бороненко (Челябинск — Москва).

Средняя оценка: -1.57

Комментарии


178. Вопрос 4.19: Согласно одной не очень правдоподобной версии, свое название ОН получил за любовь к блестящим предметам. ЕГО менее распространенное название состоит из двух слов, начинающихся на одну и ту же букву. Воспроизведите это название.

Ответ: Кокосовый краб.

Комментарий: Речь идет о раке-отшельнике “пальмовый вор”, питающемся мя-котью ко-косовых орехов.

Источник:
    1. http://ru.wikipedia.org/wiki/Пальмовый_вор
    2. http://bigpicture.ru/?p=370786

Автор: Сергей Лобачев (Нижний Новгород).

Средняя оценка: -1.86

Комментарии


179. Вопрос 5.3: Потрясённый персонаж Джона БЭнвилла сидит в автомобиле, и в какой-то момент ему кажется, что треснуло лобовое стекло. Но потом понимает, что на самом деле происходит это. Что именно?

Ответ: он плачет.

Зачёт: по слову «плач» или «слёзы» и их производным.

Комментарий: «Солнечные лучи, бившие в глаза, двоились и троились, и я подумал: уж не треснуло ли лобовое стекло, но потом, приложив руку к лицу, сообразил, что плачу».

Источник: Бэнвилл Д. Улики (http://goo.gl/Bcjo26).

Автор: Алексей Бороненко (Челябинск — Москва).

Средняя оценка: -2.08

Комментарии


180. Вопрос 4.11: Оксфордский профессор Дэ́вид Смит, образно сравнивает исчезающие руди-менты человеческой клетки с НЕЙ. Назовите ЕЁ тремя словами.

Ответ: Улыбка чеширского кота.

Комментарий: Человеческая клетка избавилась от большинства рудиментов, сохранив лишь едва заметные из них. Высказывание ученого содержит отсылку на произведение другого известного оксфордского профессора.

Источник:
    1. http://lib.rus.ec/b/395598/read
    2. http://ru.wikipedia.org/wiki/Льюис_Кэрролл

Автор: Сергей Лобачев (Нижний Новгород).

Средняя оценка: -2.12

Комментарии


181. Вопрос 6.1: Мария Э́джуорт заметила, что ЭТО не останавливается ни на миг, поэтому высшая точка – самая опасная. ЭТО изображено на одной из карт ТарО. Назовите ЭТО.

Ответ: колесо фортуны.

Зачёт: колесо судьбы.

Источник:
    1. К. Душенко "Мысли, афоризмы и шутки выдающихся женщин" - М., Эксмо, 2008.
    2. http://ru.wikipedia.org/wiki/Колесо_Фортуны_(карта_Таро)

Автор: Мишель Матвеев (Санкт-Петербург)

Средняя оценка: 0

Комментарии


182. Вопрос 6.2: По дороге в Рим на Олимпиаду 1960 года Кассиус Клей почти непрерывно молился. Дело в том, что у Клея была ОНА. Назовите ЕЁ одним словом.

Ответ: аэрофобия.

Зачёт: авиафобия.

Комментарий: Клей очень боялся лететь на самолете, но другого способа добраться не нашлось.

Источник: http://www.sports.ru/tribuna/blogs/vitalysuvorov/449732.html

Автор: Мишель Матвеев (Санкт-Петербург)

Средняя оценка: 0

Комментарии (3)


183. Вопрос 6.3: Существует легенда, что во время Семилетней войны российский генерал ТотлЕбен обратил внимание на необычайное сходство. Тем самым Тотлебен подсказал ЕМУ замысел, обернувшийся для России многочисленными бедами, а для НЕГО самого гибелью. Назовите ЕГО.

Ответ: (Емельян) Пугачёв.

Комментарий: генерал Тотлебен подметил, что участвовавший в Берлинской кампании 1760 года казак Емельян Пугачёв необычайно похож на наследника российского престола. Впоследствии поднявший бунт Пугачёв объявил себя чудом спасшимся императором Петром III.

Источник:
    1. http://ru.wikipedia.org/wiki/Тотлебен,_Готтлоб_Курт_Генрих
    2. http://www.vehi.net/istoriya/potto/kavkaz/17.html

Автор: Александр Кудрявцев (Николаев)

Средняя оценка: 0

Комментарии


184. Вопрос 6.4: Барон Ледер дважды предлагал погасить внешний долг своей страны в обмен на определённые уступки. Какие пять букв мы пропустили в предыдущей фразе?

Ответ: нарко.

Комментарий: наркобарон Карлос Ледер на рубеже 1970-1980 гг. обладал достаточным для этого финансовым могуществом. Взамен Ледер в первый раз просил прАва на свободную продажу наркотиков, а во второй раз - защиты от экстрадиции.

Источник: http://en.wikipedia.org/wiki/Carlos_Lehder#Norman.27s_Cay

Автор: Александр Кудрявцев (Николаев)

Средняя оценка: 0

Комментарии (1)


185. Вопрос 6.5: Геродот сообщает, что в войске царя Ксеркса имелась священная колесница Зевса. Комментируя это утверждение Геродота, историк Стратановский дает пояснение, в котором упоминается другое имя собственное, имеющее отношение к транспортному средству. Какое?

Ответ: АхурамАзда.

Зачёт: Мазда, Ахура Мазда, ОрмУзд, ОрмАзд.

Комментарий: Геродот отождествлял персидских богов с соответствующими греческими. В частности, зороастрийского верховного бога Ахурамазду называл Зевсом.

Источник: Геродот "История", кн.7,40, М., "Аст", 2006.

Автор: Мишель Матвеев (Санкт-Петербург)

Средняя оценка: 0

Комментарии


186. Вопрос 6.6: ИКС и ИКСОВЫЙ в вопросе - замены. Карл САган ввел термин ИКСОВЫЙ шовинизм. По его мнению, ИКСОВЫЙ шовинизм, к которому по естественным причинам склонны все люди, сильно ограничивает наши представления о жизни. А как называется характеристика, которая для ИКСа почти всегда равна четырем?

Ответ: валентность.

Комментарий: ИКС - углерод. Известный астрофизик Карл Саган отметил, что наши представления о внеземной жизни основаны на том, что она, скорее всего, тоже будет углеродной. Номер вопроса совпадает с номером углерода в таблице Менделеева :)

Источник:
    1. http://ru.wikipedia.org/wiki/Углеродный_шовинизм
    2. http://en.wikipedia.org/wiki/Carbon

Автор: Мишель Матвеев (Санкт-Петербург)

Средняя оценка: 0

Комментарии


187. Вопрос 6.7: Фотограф Альфред СтИглиц утверждал, что ОН похож на пароход и может считаться воплощением новой Америки. От какого предмета ОН получил своё название?

Ответ: утюг.

Комментарий: знаменитый небоскрёб “Утюг”, построенный в Нью-Йорке в начале XX века, имеет в сечении треугольную форму - отсюда сравнение с пароходом.

Источник: http://www.artsmia.org/get-the-picture/print/stieglitz.shtml

Автор: Александр Кудрявцев (Николаев)

Средняя оценка: 0

Комментарии


188. Вопрос 6.8: В миниатюре Ивана Матвеева, действие которой происходит в XX веке, Ахиллес ставит на ИКСа, а черепаха - на ИГРЕКа. Какие имена мы заменили на ИКС и ИГРЕК?

Ответ: Том, Джерри.

Зачёт: в любом порядке.

Комментарий: Ахиллес считает, что Том когда-нибудь догонит Джерри, а черепаха считает, что этого не произойдет. В апории Зенона Ахиллес пытается догнать черепаху.

Источник: И. Матвеев "Ахиллес и Черепаха".

Автор: Мишель Матвеев (Санкт-Петербург)

Средняя оценка: 0

Комментарии


189. Вопрос 6.9: [Ведущему: игнорировать кавычки.] Кен Уилман из курортного городка Моркам благодаря собаке разбогател на 43 тысячи фунтов. Гуляя с ней, Кен увидел, как животное принялось обнюхивать какой-то “камень”. Там, где образовался этот “камень”, когда-то, по известной легенде, побывал и ОН. А как ЕГО звали?

Ответ: Иона.

Комментарий: собака Уилмана нашла на пляже кусок амбры – ценнейшего парфюмерного сырья. Амбра зарождается в пищеварительной системе китов и обладает сильным запахом. Оборот “и ОН. А” - дополнительный намек на Иону.

Источник:
    1. http://www.bbc.co.uk/news/uk-england-lancashire-21254718
    2. http://ru.wikipedia.org/wiki/Иона_(пророк)

Автор: Александр Кудрявцев (Николаев)

Средняя оценка: 0

Комментарии


190. Вопрос 6.10: В мультфильме по мотивам русских сказок стоящий в очереди иностранец сообщает, что, по слухам, весь дворец плесень одолела. Ответьте, использовав однокоренные слова, что собираются сделать стоящие в очереди мужчины?

Ответ: рассмешить Несмеяну.

Комментарий: царевна так много плачет, что весь дворец в сырости.

Источник: мультфильм «Про Ивана-дурака», цикл «Гора самоцветов».

Автор: Александр Кудрявцев (Николаев)

Средняя оценка: 0

Комментарии


191. Вопрос 6.11: [Ведущему: Dungeons&Dragons читать как “дЭнженс энд дрЭгэнс”] В настольных ролевых играх типа Dungeons&Dragons участвуют игроки, а также мастер, который управляет игровым миром. Можно сказать, что действия мастера регулярно вступают в противоречие с убеждением, высказанным, например, в письме 1947 года. В чем состоит это убеждение?

Ответ: бог не играет в кости.

Комментарий: мастера можно считать богом игрового мира. Мастер регулярно бросает кости для имитации случайных событий. Эйнштейн неоднократно высказывал убеждение, что “бог не играет в кости”, возражая против случайности поведения частиц, которую декларирует квантовая механика.

Источник:
    1. http://ru.wikipedia.org/wiki/Dungeons&Dragons
    2. http://ru.wikiquote.org/wiki/Альберт_Эйнштейн

Автор: Мишель Матвеев (Санкт-Петербург)

Средняя оценка: 0

Комментарии


192. Вопрос 6.12: В испанской легенде рассказывается о транспортном происшествии, случившемся с набожным человеком, и чуде, совершённом Девой Марией. Можно сказать, что герой произведения 1929 года превзошёл Деву Марию в два, а, возможно, даже в четыре раза. С кем в упомянутом произведении перед этим произошла трагедия?

Ответ: с зайчиком.

Комментарий: Дева Мария даровала набожному испанцу новую ногу. Доктор Айболит пришил зайчику новые ножки, причём, сколько именно в стихотворении Корнея Чуковского не говорится.

Источник:
    1. http://saints.katolik.ru/modules.php?name=Content&pa=showpage&pid=16
    2. К.Чуковский “Айболит” (http://poetrylibrary.ru/stixiya/dobryj-doktor-ajbolit.html)

Автор: Александр Кудрявцев (Николаев)

Средняя оценка: 0

Комментарии


193. Вопрос 6.13: Согласно санитарным нормам, проведение ТАКИХ уроков в начальной школе в России запрещено. Начинающих шахматистов учат, что ТАКИЕ ПЕРВЫЕ - обычно плохо, а ТАКИЕ ВТОРЫЕ - обычно хорошо. Назовите ПЕРВЫЕ и ВТОРЫЕ в правильном порядке.

Ответ: пешки, ладьи.

Комментарий: ТАКИЕ - сдвоенные.

Источник:
    1. http://www.regobraz.ru/statia.php?nm=173
    2. Элементарные основы шахмат.

Автор: Мишель Матвеев (Санкт-Петербург)

Средняя оценка: 0

Комментарии


194. Вопрос 6.14: [Ведущему: Indian Opinion читать как “Индиан опИнион”] В 1905 году в газете Indian Opinion были опубликованы рассказы «Бог любит правду», «Нить жизни» и «Алчность». Назовите фамилии автора этих рассказов и их переводчика.

Ответ: Толстой, Ганди.

Зачёт: в любом порядке.

Комментарий: Махатма Ганди был приверженцем толстовства.

Источник: http://ru.wikipedia.org/wiki/Бог_правду_видит,_да_не_скоро_скажет

Автор: Александр Кудрявцев (Николаев)

Средняя оценка: 0

Комментарии


195. Вопрос 6.15: ОН совершил путешествие от села Деденёво до Астрахани, но там был захвачен отрядом Степана Разина и уничтожен. Какими тремя словами называет ЕГО Наум Синдаловский?

Ответ: прадедушка русского флота.

Комментарий: это корабль “Орёл”, построенный при Алексее Михайловиче. Как известно, при Петре I, сыне Алексея Михайловича, был построен ботик, который называют дедушкой русского флота.

Источник: Н. Синдаловский “Санкт-Петербург – история в преданиях и легендах” (http://flibusta.net/b/202143/read).

Автор: Мишель Матвеев (Санкт-Петербург)

Средняя оценка: 0

Комментарии


196. Вопрос 6.16: [Ведущему: politique potentielle читать как “политИк потонсьЕл”] Этот человек, учившийся с 1949 по 1953 годы во французском вузе, считал себя политиком большого потенциала, и, к несчастью, реализовал этот потенциал. Под каким именем он стал известен?

Ответ: Пол Пот.

Комментарий: сокращение от французского "politique potentielle".

Источник: http://en.wikipedia.org/wiki/Pol_Pot

Автор: Мишель Матвеев (Санкт-Петербург)

Средняя оценка: 0

Комментарии


197. Вопрос 6.17: Один представитель современной профессии рассказал, что множество людей пообещало ему принести своего первенца в жертву богу-крокодилу. Дело в том, что упомянутый человек не только разработал свой продукт, но и сопроводил его юмористическим ЭТИМ. Назовите ЭТО двумя словами.

Ответ: лицензионное соглашение.

Зачёт: пользовательское соглашение.

Комментарий: рассказчик написал программу, в условия пользования которой поместил требование принести своего первенца в жертву богу-крокодилу. Естественно, лицензионное соглашение пользователи, как обычно, принимали, не читая.

Источник:
    1. http://www.shorts.ru/s.php/459343126.htm
    2. http://softoor.narod.ru/content.html

Автор: Мишель Матвеев (Санкт-Петербург)

Средняя оценка: 0

Комментарии


198. Вопрос 6.18: [Ведущему: Dragonlance читать как “дрЭгэн ланс”] Цитата из книги, принадлежащей к эпопее Dragonlance: "ТАнис наклонился к ручью и принялся жадно пить." А какое полное имя носит этот персонаж эпопеи?

Ответ: Танталас.

Зачёт: Танталос, Тантал, Танталис [Полуэльф].

Комментарий: процитированный эпизод перекликается с мифом о Тантале.

Источник:
    1. М. Уэйс “Драконы осенних сумерек” (http://tululu.org/read22529/56/)
    2. http://www.dragonlance.ru/world/text/heroes/

Автор: Мишель Матвеев (Санкт-Петербург)

Средняя оценка: 0

Комментарии


199. Вопрос 6.19: Энциклопедия "Альфа и Омега" называет ИКС двенадцатым по величине в мире, если считать максимальную величину в течение года. Минимальная же его величина в два с лишним раза меньше максимальной. А согласно фразеологическому словарю Михельсона, ИКС - это смутное состояние души. Какое слово мы заменили на ИКС?

Ответ: Чад.

Комментарий: размер озера Чад сильно зависит от сезона.

Источник:
    1. Энциклопедия "Альфа и Омега", раздел “Озёра”
    2. http://onlineslovari.com/malyiy_akademicheskiy_slovar/page/chad.75289/

Автор: Мишель Матвеев (Санкт-Петербург)

Средняя оценка: 0

Комментарии


200. Вопрос 6.20: Журналист пишет, что увидел Александра Поветкина, который выглядел прекрасно, находился в оптимальной форме, а роль ЭТОГО играл небольшой фингал на правом глазу. ЭТО имеет сравнительно низкую температуру. Назовите ЭТО.

Ответ: пятно на солнце.

Зачёт: солнечное пятно, солнечные пятна, пятна на солнце.

Комментарий: солнечные пятна имеют более темный цвет, чем окружающая поверхность, поскольку обладают более низкой температурой. Журналист имел в виду выражение “и на солнце бывают пятна”, означающее, что и у самого безупречного найдутся дефекты. Слова “увидел” и “выглядел” употреблены не случайно.

Источник:
    1. http://www.sport-express.ru/newspaper/2007-10-27/8_1/
    2. http://ru.wikipedia.org/wiki/Солнечные_пятна

Автор: Мишель Матвеев (Санкт-Петербург)

Средняя оценка: 0

Комментарии


201. Вопрос 6.21: В триллере “Дуэль” водитель-убийца прикручивает несколько ИХ к бамперу своего грузовика, чем-то напоминая кровожадного индейца. Назовите ИХ двумя словами.

Ответ: номерные знаки.

Зачёт: автомобильные номера, номера машин, номера автомобилей.

Комментарий: водитель грузовика прикреплял на бампер номера уничтоженных им машин, которые можно считать аналогом снятых с врагов скальпов.

Источник: фильм «Дуэль» (реж. – Стивен Спилберг).

Автор: Александр Кудрявцев (Николаев)

Средняя оценка: 0

Комментарии


202. Вопрос 6.22: Цитата. "Пылкая фантазия авторов героической фэнтези находит для них одно-единственное и неизменное предназначение, а именно: где-нибудь в районе третьей главы (или на десятой минуте фильма) ворваться в комнату, по очереди атаковать героя и быть уложенными на месте." Конец цитаты. Назовите процитированное произведение. Назовите процитированное произведение.

Ответ: "Стража! Стража!"

Комментарий: речь о стражниках. Повтор в конце вопроса - намек на двойное название. Цитата довольно характерна для стиля Пратчетта.

Источник: Т. Пратчетт "Стража! Стража!"

Автор: Мишель Матвеев (Санкт-Петербург)

Средняя оценка: 0

Комментарии


203. Вопрос 6.23: Человек, которого придворный английского короля Карла Первого вызвал на дуэль, явился на поединок с водяным пистолетом, однако был убит выстрелом в голову. Какое существительное мы пропустили в предыдущей фразе?

Ответ: шут.

Зачёт: карлик.

Комментарий: человек, вызванный на дуэль придворным шутом короля, не отнесся к этому серьезно, а зря.

Источник:
    1. http://en.wikipedia.org/wiki/Jeffrey_Hudson
    2. http://www.rulit.net/books/the-book-of-the-dead-read-250231-56.html
    3. http://thehumanmarvels.com/133/jeffrey-hudson-lord-minimus/dwarfism

Автор: Александр Кудрявцев (Николаев)

Средняя оценка: 0

Комментарии


204. Вопрос 6.24: Балетмейстер Ролан Пети, женатый на балерине Зизи Жанмер, говорил, что у них получилось ЭТО длиною в жизнь. Комплекс упражнений для придания ягодицам идеальной формы образно назван ЭТИМ для ягодиц. Назовите ЭТО.

Ответ: па-де-дё.

Зачёт: па-де-де.

Комментарий: известный танцор и балетмейстер Ролан Пети уподобил свою жизнь в паре с Зизи Жанмер па-де-дё – балетному танцу для двоих исполнителей. Человеческое тело, как известно, укомплектовано двумя ягодицами, движения которых при выполнении упражнений напоминают своеобразный танец.

Источник:
    1. Ролан Пети: Между прошлым и будущим (документальный фильм), 00:18:15;
    2. http://www.shape.ru/pa-de-de-dlya-yagodits.html

Автор: Александр Кудрявцев (Николаев)

Средняя оценка: 0

Комментарии


205. Вопрос 6.25: Недавно произошедшее в НЁМ событие заставило одного комментатора предположить, что там имеется гравитационная аномалия. Ведь предыдущее аналогичное событие тоже произошло в НЁМ. Назовите ЕГО.

Ответ: Донецк.

Комментарий: в феврале 2014 года Рено Лавиллени в Донецке побил мировой рекорд по прыжкам с шестом. Предыдущий рекорд был также установлен в Донецке Сергеем Бубкой, жизнь и карьера которого связаны с этим городом.

Источник: http://lenta.ru/comments/news/2014/02/15/nobubka/

Автор: Мишель Матвеев (Санкт-Петербург)

Средняя оценка: 0

Комментарии


206. Вопрос 6.26: ОНИ стали названием сайта, созданного для любителей интеллектуальных игр, в частности, сканвордов и кроссвордов. Назовите ИХ двумя словами.

Ответ: серые клеточки.

Зачёт: серые клетки.

Комментарий: для интеллектуальных игр необходимо задействовать серые клеточки мозга. В кроссвордах и сканвордах можно встретить серые клетки.

Источник: http://www.graycell.ru/

Автор: Александр Кудрявцев (Николаев)

Средняя оценка: 0

Комментарии


207. Вопрос 6.27: Рост человека зависит не только от генов. Например, средний рост жителей ПЕРВОЙ на 7 сантиметров меньше, чем жителей ВТОРОЙ. Назовите ПЕРВУЮ и ВТОРУЮ в правильном порядке.

Ответ: Северная Корея, Южная Корея.

Зачёт: КНДР/Корейская Народно-Демократическая Республика, Республика Корея.

Комментарий: генофонд у корейцев практически одинаковый, а рост отличается. По мнению ученых, причиной этого является постоянное недоедание жителей Северной Кореи на протяжении нескольких поколений.

Источник:
    1. http://ru.wikipedia.org/wiki/Рост_человека
    2. http://www.economist.com/node/21542185

Автор: Мишель Матвеев (Санкт-Петербург)

Средняя оценка: 0

Комментарии (1)


208. Вопрос 6.28: Согласно демотиватору, с каждым годом всё сложнее пить виски ДВА СЛОВА ПРОПУЩЕНО. В школьные годы Николай Пундик играл в народном театре со Снегурочками ДВА СЛОВА ПРОПУЩЕНО роль Деда Мороза. Какие два слова, начинающиеся на одну и ту же букву, мы пропустили в обоих случаях?

Ответ: старше себя.

Комментарий: старый виски и найти, и приобрести непросто. В упомянутых спектаклях Дед Мороз оказывался младше Снегурочки.

Источник:
    1. http://www.funkyimg.com/i/FEBD.jpg
    2. http://www.048.ua/news/35533

Автор: Александр Кудрявцев (Николаев)

Средняя оценка: 0

Комментарии


209. Вопрос 6.29: ИКС в вопросе - замена. Когда отец Луиса Бунюэля прибыл служить на Кубу, внимание начальства привлек его формуляр. В итоге Бунюэль-старший, в отличие от прибывших с ним друзей, остался жив. Можно сказать, что это произошло благодаря его ИКСу. Как называется специалист, предметом изучения которого является ИКС?

Ответ: графолог.

Комментарий: ОН – почерк. Увидев каллиграфический почерк отца Бунюэля, его решили оставить при штабе писарем, благодаря чему он и остался в живых.

Источник: Л.Бунюэль “Мой последний вздох” (http://flibusta.net/b/71646/read)

Автор: Александр Кудрявцев (Николаев)

Средняя оценка: 0

Комментарии


210. Вопрос 6.30: "Если бы у нас действительно было столько солдат, мы бы выиграли эту войну!" Известная женщина сказала эту фразу в 1939 году, увидев... что?

Ответ: (фильм) "Унесенные ветром".

Комментарий: Маргарет Митчелл показалось, что соотношение сил в фильме было гораздо более оптимистичным для южан, чем в действительности.

Источник: К. Душенко "Мысли, афоризмы и шутки выдающихся женщин" - М., Эксмо, 2008.

Автор: Мишель Матвеев (Санкт-Петербург)

Средняя оценка: 0

Комментарии


211. Вопрос 6.31: В одном американском шоу герой-вампир страдает навязчивой потребностью подсчитывать предметы. В оригинале его называют тремя словами, одно из которых – немецкая приставка, а два других - омонимы. Напишите эти три слова.

Ответ: Count von Count [кАунт фон кАунт].

Зачёт: Count fon Count; Каунт фон Каунт.

Комментарий: в английском языке слово «Count» означает и «граф», и «считать». Герой «Улицы Сезам». Граф-вампир - отсылка к Дракуле.

Источник: http://ru.wikipedia.org/wiki/Граф_фон_Знак

Автор: Александр Кудрявцев (Николаев)

Средняя оценка: 0

Комментарии (1)


212. Вопрос 6.32: Когда Людовик Жюли перешел в "Барселону" и впервые явился в клуб, он постоянно ловил на себе странные взгляды. Президент клуба объяснил ему ситуацию и велел никогда больше не ДЕЛАТЬ ЭТОГО. Многие женщины тоже стараются не ДЕЛАТЬ ЭТОГО. Что мы заменили на ДЕЛАТЬ ЭТО?

Ответ: носить белое.

Зачёт: надевать/носить белое/белую одежду/белые цвета/белую футболку.

Комментарий: принципиальный соперник "Барселоны" - мадридский "Реал", традиционный цвет которого - белый. По распространенному мнению, белая одежда полнит.

Источник: http://www.armsport.am/ru/news/2013/05/15/жюли/305291

Автор: Мишель Матвеев (Санкт-Петербург)

Средняя оценка: 0

Комментарии


213. Вопрос 6.33: Некоторые исследователи творчества Толкиена отмечают, что в его мире фигурируют мудрые эльфы, воинственные люди, искусные мастера гномы, хоббиты-земледельцы и грязные, мерзкие орки, с которыми остальные народы Средиземья не желают иметь дела. С кем эти исследователи сравнивают орков?

Ответ: с неприкасаемыми.

Зачёт: с париями.

Комментарий: эльфы - брахманы, люди - кшатрии, гномы и хоббиты - вайшьи и шудры, орки - неприкасаемые.

Источник: http://orchina.narod.ru/ingar.htm

Автор: Мишель Матвеев (Санкт-Петербург)

Средняя оценка: 0

Комментарии


214. Вопрос 6.34: Ростислав Чебыкин написал веселое продолжение этой короткой стихотворной истории. Один из героев оказывается ученым-экспериментатором, и продолжает эксперименты, рискуя собой, а другой герой остается отшельником, который агрессивно реагирует на все попытки вторжения в его среду обитания. А из какой страны родом этот экспериментатор?

Ответ: из Греции.

Комментарий: продолжение истории "Едет Грека через реку". Ученый Грека, изучая поведение рака, поочередно сует в реку одну и другую руку, затем ногу и даже голову. Слово “отшельник” - намек на рака-отшельника.

Источник: Р. Чебыкин "Про Греку" (http://muzbar.ru/tabs/Filigon/Pro_Greku/)

Автор: Мишель Матвеев (Санкт-Петербург)

Средняя оценка: 0

Комментарии


215. Вопрос 6.35: На одной картине дети сидят за столом и, затаив дыхание, наблюдают за действиями своей сестры. Названием картины стало устойчивое словосочетание, которое, по мнению автора вопроса, таковым не является. Напишите это название.

Ответ: «Карточный домик».

Комментарий: затаив дыхание - потому что неосторожного вздоха достаточно, чтобы всё развалилось. Автор вопроса считает устойчивое словосочетание «карточный домик» неустойчивым.

Источник: http://www.wikipaintings.org/ru/zinaida-serebriakova/house-of-cards-1919

Автор: Александр Кудрявцев (Николаев)

Средняя оценка: 0

Комментарии


216. Вопрос 6.36: Один американец перед тем, как покончить с собой, оставил записку. Он писал, что откажется от самоубийства, если на пути к мосту Золотые Ворота встретит хотя бы одного человека, который СДЕЛАЕТ ЭТО. В древнескандинавском языке имелось пять слов со значением “убивать” и лишь одно, означающее “ДЕЛАТЬ ЭТО”. Какое слово мы заменили на ДЕЛАТЬ ЭТО?

Ответ: улыбаться.

Зачёт: улыбнуться.

Комментарий: бедняге так никто и не улыбнулся. А древние скандинавы были весьма суровы. Надеемся, что после этого тура у вас осталось хорошее настроение.

Источник:
    1. http://www.newyorker.com/archive/2003/10/13/031013fa_fact?currentPage=all
    2. http://www.yorku.ca/inpar/language/English-Old_Norse.pdf

Автор: Александр Кудрявцев (Николаев)

Средняя оценка: 0

Комментарии

Вход в систему